Vous êtes sur la page 1sur 84

http://tailieuso.

com/

GMAT
Reading comprehension practice

http://tailieuso.com/

On May 5th, 1997, the European edition of Business Tech Magazine led with Hoffmans cover story "Internet Communities: How They're Shaping Electronic Commerce". This cover story highlights the extent to which the term virtual community has become almost synonymous with various forms of group-CMCs (computer mediated communication), including email-list forums, chat-systems such as IRC, web-based discussion areas and usenet news-groups. There was no debate in the Business Tech Magazine article as to whether the group-CMC discussions are really 'communities', rather how community as opposed to content can be used to encourage people to return to a particular part of cyberspace for commercial gain. In a similar vein, Simpson and Armstrong in "Internet Gain" argue that ignoring virtual communities would be a great loss of a marketing tool for businesses. They define virtual communities as computer mediated space where there is an integration of content and communication with an emphasis on member-generated content. Not all virtual community commentators agree with the Spartan position taken by Hoffman. Rheingold, one of the prime popularizers of the term virtual community, provides us with a more emotive definition in his book The Virtual Community: Homesteading on the Electronic Frontier. According to Rheingold, "virtual communities are social aggregations that emerge from the Net when enough people carry on those public discussions long enough, with sufficient human feeling, to form webs of personal relationships in cyberspace". Rheingold's definition is extremely popular and has been quoted in many discussions about virtual communities. As discussed below, for social scientists, particularly sociologists, Rheingold's definition raises many issues, especially concerning the notion of community. This is because Rheingold argues via a variety of analogies from the real world such as homesteading that virtual communities are indeed new forms of "community". In fact, Rheingold implies that virtual communities are actually "a kind of ultimate flowering of community". Moreover, Rheingold maintains that whenever computer mediated communications technology becomes available, people inevitably
2

http://tailieuso.com/

create communities with it. Rheingold can thus be labeled as a technological determinist as he holds that there is a predictable relationship between technology and people's behavior. The debate over the validity of Rheingold's position has raised doubts about the existence of virtual communities and the appropriate use of the term. Weinreich claims that the idea of virtual communities must be wrong because community is a collective of kinship networks that share a common geographic region, a common history, and a shared value system, usually rooted in a common religion. In other words, Weinreich rejects the existence of virtual communities because group-CMC discussions cannot possibly meet his definition. In Weinreich's view, anyone with even a basic knowledge of sociology understands that information exchange in no way constitutes a community.

1. The primary purpose of the passage is to (A) suggest an alternate definition for the term virtual communities (B) challenge the validity of group-CMCs in the virtual community (C) discuss whether group-CMCs constitute real communities (D) present two opposing hypotheses and presents research and evidence to support them (E) emphasize the unsuitability of traditional definitions of community in light of the recent establishment of virtual communities on the Net The best answer is C. The passage begins by presenting the viewpoint of those that believe that group-CMCs are true communities and continues by presenting an opposing view. D is incorrect because definitions, not hypotheses are discussed in the passage.

2. According to Simpson and Armstrong, virtual communities (A) are not as effective as content in encouraging people to return to a particular part of the Internet. (B) emphasize attracting new members through their use of absorbing content
3

http://tailieuso.com/

(C) are not really communities, but simply group-CMC discussions (D) has become almost synonymous with various forms of group-CMCs (E) are an invaluable marketing tool for businesses. The best answer is E. According to Simpson and Armstrong ignoring virtual communities would be a loss. In other word, virtual communities are invaluable. The word invaluable means very useful.

3. Weinreich rejects Rheingolds inclusion of virtual communities in the definition of communities for all of the following reasons EXCEPT (A) virtual communities do not usually exchange information (B) virtual communities do not usually share the same territory (C) virtual communities do not usually share values (D) virtual communities do not usually share a common history (E) virtual communities are not usually rooted in a common religion The best answer is A. The question asks you to identify what does NOT stop Weinreich from considering a virtual community a real community. An exchange of information is not problematic for Weinreich. 4. The author says there was no debate in the Business Tech Magazine article to emphasize which of the following points? (A) It is not content but community that can be used to entice the public to go back to a commercial website (B) It is not community but content that can be used to entice the public to go back to a commercial website (C) It would be a great loss of a business marketing tool if virtual communities were ignored. (D) There are various forms of group-CMCs, including email-list forums, chatsystems, web-based discussion areas and usenet news-groups. (E) Business Tech Magazine had already made the assumption that groupCMCs are communities. The best answer is E. The passage discusses whether group-CMCs are communities. Business Tech Magazine does not debate this issue because it has already made the assumption that they are.

http://tailieuso.com/

Take a very commonplace, often discussed and critical topic: Are we detecting a greenhouse effect, and related to this, is it exacerbated by "homogenic factors," i.e., human actions? Most would be inclined to give a positive answer to both of these questions. But, if pushed, what would be the evidence, and how well grounded would it be for such affirmations? Within scientific communities and associated scientifically informed circles, the answers have to be somewhat more ambiguous, particularly when rigorous questions concerning evidence are raised. Were scientific truth to be a matter of consensus, and some argue that scientific truth often turns out to be just that, then it is clear that there is beginning to be a kind of majority consensus among many earth science practitioners that the temperature of the Earth, particularly of the oceans, is indeed rising and that this is a crucial indicator for a possible greenhouse effect. Most of these scientists admit that the mean oceanic temperature has risen globally in the last several decades. But this generalization depends upon how accurate measurements may be, not just for samples, but also for the whole Earth. Hot spots, for example the now four year old hot spot near New Guinea which is part of the El Nio cycle, does not count by itself because it might be balanced by cold spots elsewhere. And the fact of the matter is that "whole earth measurements" are still rare and primitive in the simple sense that we simply do not have enough thermometers out. Secondly, even if we had enough thermometers, a simply synchronic whole earth measurement over three decades is but a blip in the diachronic history of ice age cycles over the last tens of thousands of years. Thirdly, even if we know that the earth is now heating up, has an ever increasing ozone hole, and from this strange weather effects can be predicted, how much of this is due to homogenic factors, such as CFCs, CO2 increases, hydrocarbon burning, and the like? Is it really the case, as Science magazine claimed in l990, "24% of greenhouse encouraging gases are of homogenic origin"? 1. In this passage the author is primarily interested in (A) whether scientific truths are simply a matter of consensus (B) determining how well established the greenhouse effect is and to what degree it is worsened by human actions (C) whether the hot spot El Nio is balanced elsewhere by cold spots (D) determining if most scientists would be inclined to give a positive answer to the question of whether there is a greenhouse effect and if it is worsened by human actions (E) making a simple synchronic whole earth measurement more than a blip in the diachronic history of Ice Age cycles over the last tens of thousands of years.

http://tailieuso.com/

The best answer is B. The author questions the claim that there is indeed a greenhouse effect that is made worse by human actions. (A) is too general an answer, while (D) is too specific. (D) is wrong because it is probing whether scientists agree, not whether there the phenomenon actually exists.

2. The author of the passage would be most likely to agree with which of the following statements about the greenhouse effect? (A) 24% of greenhouse encouraging gases are of homogenic origin. (B) there is a greenhouse effect that is exacerbated by homogenic factors. (C) The ozone hole is increasing due to homogenic factors, such as CFCs, CO2 increases, hydrocarbon burning, and the like. (D) One can determine if mean oceanic temperatures have risen globally in the last several decades only if measurements of ocean temperatures are precise. (E) Hot spots, such as the El Nio cycle, should not be counted as a factor in the greenhouse effect. The best answer is D. Scientists are basing their claims on global warning on rising ocean temperatures. One can tell if temperatures have in fact risen only by measuring them correctly.

3. It can be inferred from the passage that (A) we cannot be certain that strange weather effects are a result of the earth heating up and an ever increasing ozone hole (B) the greenhouse effect is the most widely discussed topic in the scientifically informed circles (C) If the temperature of the oceans has ceased to rise at an ever increasing rate, then the rate of global warming has increased (D) strange weather effects have been shown to be due to the diachronic effects of hydrocarbon burning and not to increases in CFC. (E) Strange weather effects are caused by the increase use of CFCs, CO2, and similar gasses. The best answer is A. The author is questioning the cause and effect relationship between the increasingly large ozone hole and global warming, as well as cause and effect relationship between global warming and strange weather effects. 4. The authors claim that, a simply synchronic whole earth measurement over three decades is but a blip in the diachronic history of ice age cycles over the last tens of thousands of years would be strengthened if the author

http://tailieuso.com/

(A) indicated the minimum number of thermometers necessary for a whole earth measurement (B) described the factors that precipitated the start of a new ice age (C) compare synchronic whole earth measurements with diachronic whole earth measurements (D) proved that the mean number of years required to detect significant changes in weather patterns is greater than thirty (E) specified the exact location and quantity of thermometers placed by scientists around the globe The best answer is D. If one knows that change can be detected only after much more than thirty years, then measurements taken over a thirty year period is insignificant

http://tailieuso.com/

An Australian group named Action Council on Smoking and Health (ACSH) has recently lobbied to make warnings on cigarette packets more graphic. The council proposed that striking visual photos of diseased organs should be put on at least 50% of outside packaging, in conjunction with health warnings outlining smoking hazards enumerated in a separate leaflet placed inside the cigarette packet. The ACSH claim that bland and ineffectual warnings like "Smoking is a health hazard" currently found on cigarette packets are not nearly sufficient.

Substituting those inadequate admonitions with explicit photos will provide a powerful visual stimulus to help smokers relinquish their habit. The current cautions on cigarette packets have little or no impact on smokers who have grown immune to the warnings that focus on abstract tobacco related risks and illnesses from which smokers can easily disassociate themselves. The proposed new tactics would concentrate on the perspective of the individual smoker through a demonstration of what is occurring in his body each time he reaches for a cigarette, rather than a generic cautionary word of advise.

The ACSH cited the results of recent studies conducted by psychologists at McKean University confirming that evidence related to one's own experience is more effective at influencing future behavior than a presentation of facts and figures. An further rationale for the addition of pictures to cigarette packages is the finding that smokers handle their packets 20-30 times a day, on average, thus, if graphic pictures on cigarette packets were introduced, smoker would have 20-30 chances to face the harsh reality of what damage they are doing to themselves each time they light up.

http://tailieuso.com/

Even more essential than the pictures on the outside label, ACSH strongly advocate including warnings and helpful information in a leaflet inserted into the packet of cigarettes. Even an analgesic, ACSH adds, found in every bathroom cabinet has all possible side effects enumerated in the insert. How much more imperative is it then when the substance in question is tobacco, a dried weed that contains highly noxious nicotine that society still accepts even though it kills one of every two of its users.

Fundamentally, what is at stake here is consumer rights. Smokers should know what substances they are inhaling, and what damage they are inflicting to their bodies, though surprisingly, even today, many do not. For this reason alone, the recommendation for more graphic pictures and warnings on cigarette packets, which many seem excessive, is being seriously considered.

1. It can be inferred from the passage (A) That cigarette manufacturers would comply with regulations ordering them to add graphic pictures of diseased organs to their outside packaging. (B) That society will not continue to condone smoking if it is proven even more dangerous than was previously assumed. (C) That smoking cigarettes causes damage to the internal organs of the body. (D) That if the written warnings were less bland and ineffectual, smokers would not take more notice of them. (E) That smokers look at their cigarette packages each time they take out a cigarette. The best answer is C. We do not have information about (A) (B) and (D) from the passage. (E) is incorrect because the passage claims that smokers have an opportunity to look at their cigarette packages, not that they actually do.
9

http://tailieuso.com/

2. The author cites studies conducted at McKean University to account for why

(A) A presentation of facts and figures is more effective at influencing future behavior than evidence related to one's own experience. (B) A presentation of facts and figures is less effective at influencing future behavior than evidence related to one's own experience. (C) Evidence related to one's own experience has a more long-lasting effect than future behavior. (D) The ACSH claim that graphic visual pictures of diseased organs would not be more effective than stating facts about the consequences to the body of long-term smoking. (E) The ACSH claim that graphic visual pictures of diseased organs would not be less effective than stating facts about the consequences to the body of long-term smoking.

The best answer is E. The author mentions the study as evidence presented by the ACSH to back their claim that visual pictures would be more effective than the present warning found on cigarette boxes. 3. Which of the following, if true, would be most useful in supporting the claims made by the ACSH? (A) There is firm evidence that information communicated in a textual format is more convincing than the same information conveyed in the form of visual depictions. (B) There is firm evidence that information conveyed in the form of visual depictions is more convincing than the same information communicated in a textual format.

10

http://tailieuso.com/

(C) A study of over 3000 individuals shows a statistically significant relationship between levels of nicotine in cigarettes and pulmonary damage. (D) A study of over 3000 individuals shows a statistically significant relationship between smoking and pulmonary damage. (E) A survey reveals that 79% of smokers look at their cigarette packages when taking out a cigarette.

The best answer is B. If smokers were more convinced of the dangers of smoker by pictures than by text, they would be more likely to be influenced by the pictures that the ACSH is proposing. 4. The passage does NOT state which of the following about smoking warnings. (A) Current graphic warnings are effective. (B) The addition of graphic warnings would be an impetus to smokers to relinquish their habit. (C) Current written warnings are not adequate. (D) Current written warnings are not effective. (E) Current written warnings are not as exhaustive as those that accompany common analgesics.

The best answer is A. The passage does not say that current graphic warnings are effective since there are currently no graphic warnings, only written ones.

11

http://tailieuso.com/

Gene therapy offers a new treatment paradigm for curing human disease. Rather than altering the disease phenotype by using agents that interact with gene products, or are themselves gene products, gene therapy can theoretically modify specific genes resulting in disease cure following a single administration. Initially gene therapy was envisioned for the treatment of genetic disorders, but is currently being studied for use with a wide range of diseases, including cancer, peripheral vascular disease, arthritis, Neurodegenerative disorders and other acquired diseases. Certain key elements are required for a successful gene therapy strategy. The most elementary of these is that the relevant gene be identified and cloned. Upon completion of the Human Genome Project, gene availability will be unlimited. Once identified and cloned, the next consideration must be expression of the gene. Questions pertaining to the efficiency of gene transfer and gene expression remain at the forefront of gene therapy research, with current debates revolving around the transfer of desired genes to appropriate cells, and then to obtaining sufficient levels of expression for disease treatment. With luck, future research on gene transfer and tissue-specific gene expression will resolve these issues for the majority of gene therapy protocols. Other important considerations for a gene therapy strategy include a sufficient understanding of the pathogenesis of the targeted disorder, potential side effects of the gene therapy treatment, and a more in depth understanding of the target cells which are to receive gene therapy. Gene transfer vector is the mechanism by which the gene is transferred into a cell. Currently there are at least 150 clinical gene therapy protocols worldwide. Since the approval process for these protocols is not as public outside the U.S., it is difficult to ascertain the exact number of worldwide protocols. As of December 1995, 1024 patients had been treated with either a gene transfer or gene therapy protocol. Much controversy exists regarding how many of these patients have benefited from their gene therapy, and no one has yet been cured. Public controversy in the field of human gene therapy is driven by several factors. Ordinary citizens as well as scientists easily understand the enormous potential of gene therapy, but the former may not appreciate all the pitfalls and uncertainly that lie in the immediate future. The financial interests of biotechnology firms and, some have asserted, the career interests of some gene therapists have encouraged extravagant, or at least overly optimistic public statements about contemporary gene therapy. In spite of the proliferation of protocols, the actual number of patients treated remains small, and only one genuinely controlled study of human gene therapy has been published as of this date.

12

http://tailieuso.com/

1. In the passage, the author anticipates which of the following as a possible obstacle to the introduction of gene therapy to mainstream medicine? (A) Overly optimistic public statements given by scientists who have a vested interest (B) The general publics difficulty in grasping gene therapys vast potential. (C) Unchecked financial interests of biotechnology firms. (D) The relatively small number of controlled studies of human gene therapy published as of this date. (E) Hazards of which the general public is currently unaware. The best answer is E. The passage states that both ordinary citizens and scientists understand potential of gene therapy, but the former, i.e. the general public may not appreciate all the pitfalls and uncertainty that lie in the immediate future. 2. All of the following are mentioned in the passage as elements that are required for a successful gene therapy strategy EXCEPT: (A) Identifying the relevant gene (B) Expressing the relevant gene (C) Determining the side effects of the relevant gene (D) Understanding of the pathogenesis of the targeted disorder (E) Gaining and a more in depth understanding of the target cells which are to receive gene therapy. The best answer is C. One must determine the side effects of the relevant gene therapy treatment, and not of the relevant gene itself. 3. The authors attitude toward the gene therapy as a future cure for cancer, peripheral vascular disease, arthritis, Neurodegenerative disorders and other acquired diseases is (A) Indifference (B) Disapproval (C) Amusement (D) Cautious optimism (E) Censure The best answer is D. In the opening sentence, the author optimistically states that gene therapy offers a new treatment paradigm for curing human disease. However, in the fifth paragraph the author mentions some of the problems.

4. The primary function of the fifth paragraph is to


13

http://tailieuso.com/

(A) Explain effects (B) Recommend actions (C) Identify problems (D) Evaluate solutions (E) Warn of consequences The best answer is C. The fifth paragraph is devoted to identifying various problems in the field today, such as the exaggerated claims made by biotechnological firms with vested financial interests.

Unlike the United States with its generalissimo politicians - Washington, Jackson, Grant, and Eisenhower- the martial arts have been conspicuously absent from Canadian politics. The exception to the rule is former Prime Minister Pierre Elliot Trudeau, who became the first Canadian leader to bring a gunslinger ethos to Canadian politics. Trudeau introduced Canada to the refined art of single combat; it was the politics of doing it my way. Singlecombat confrontation implied much more than the renegade in power did, and far less than the tricks of William Lyon Mackenzie King, prime minister intermittently between 1921 and 1948. Trudeaus unique background prepared him for the role of authoritarian leader he would assume later in life. Born on October 18, 1919, Trudeau lived in French-speaking Montreal, but heard English at home from his mother, making it easy for the young politician to appeal to all sectors of Canada, a bilingual country. As a young man, he walked and cycled through Europe, finding himself on occasion on the wrong side of the bars in foreign jails. By 1940, Trudeau entered the law faculty at the University of Montreal. As a student he enlisted in the Canadian Officers Training Corps, where he was given a commission as a lieutenant, a rank he held until his retirement in 1947. Trudeau, a renowned sportsman, held a brown belt in karate, knew how to skin dive and could descend 150 feet off a cliff with ease. He continued performing flamboyant physical feats even in later life as Canadas fifteenth prime minister, astounding Canadians with his prowess. The publics adoration made it possible for him to practice his personal brand of do it my way politics, initiating profound and long-lasting changes to his country. Other leaders would never have undertaken to deal with such taboo issues as divorce, abortion and homosexuality matters likely to infuriate conservative Canada from coast to coast. Even the powerful Mackenzie King dared not touch any of the three, though Trudeau tackled them together in an omnibus bill as Minister of Justice under Lester B. Pearson. His reason for loosening legislation on these issues was, as he put poetically put it, "The state has no business in the bedrooms of the nation." The myths-makers have it that this
14

http://tailieuso.com/

was Trudeau's first deliberate gun slinging move, performed with the ultimate goal of attaining national leadership. Contrary to popular belief, Trudeau had no leadership aspirations at the time; all he had was a passion for combat that eclipsed other religious considerations. Trudeau instigated far-reaching changes in legislation governing divorce, abortion and homosexuality that have had a major impact on Canada, shaping the country into what it is today. 1. The primary focus of the passage is on which of the following? (A) Comparing two Canadian prime ministers and contrasting their personal style of leadership (B) Describing the leadership style of one of Canadas prime ministers (C) Evaluating the success of the leadership style of one of Canadas prime ministers (D) Summarizing the contribution of one of Canadas prime ministers (E) Tracing the long-term impact of legislation put forward by one of Canadas prime ministers The best answer is B. The passage describes the doing it my way type leadership style of former Prime Minister Pierre Elliot Trudeau. 2. It can be inferred from the passage that former Prime Minister William Lyon Mackenzie King (A) was opposed to abortion and put forward legislation making it illegal (B) was opposed to abortion but did not put forward legislation making it illegal (C) was in favor of abortion and put forward legislation making it legal (D) was in favor of abortion but did not put forward legislation making it legal (E) did not put forward legislation making abortion legal The best answer is E. The passage states that Mackenzie King dared not touch the issue of abortion, so it can be inferred from the passage that he did not put forward any abortion legislation. The passage provides no information on what Mackenzie Kings opinions were on the subject. 3. The author of the passage mentions Trudeaus accomplishments in sports primarily in order to (A) Explain the source of Trudeaus physical stamina (B) Illustrate that he had earned the adoration of the Canadian public (C) Contrast it to his personal brand of do it my way politics (D) Provide one reason why he was able to single-handedly push through legislation that should normally have shocked conservative Canada (E) Provide one reason why he was able to single-handedly thwart legislation that should normally have shocked conservative Canada The best answer is D. One of the ways Trudeau won the hearts and minds of the Canadian public was by astounding them with his physical prowess.

15

http://tailieuso.com/

Because the public was so awed by Trudeau as an individual, he was able to pass legislation considered progressive. 4. The quotation "The state has no business in the bedrooms of the nation," is most probably used to (A) present the opinion that the state should have less of a say in issues that are essentially not public matters (B) present the opinion that the state should have less of a say in issues that are essentially public matters (C) present the opinion that the state should have more of a say in issues that are essentially not personal matters (D) provide an example of Pierre Elliot Trudeaus flamboyant style that he used to captivate the Canadian public (E) contrast Pierre Elliot Trudeaus flamboyant style with that of Lester B. Pearson The best answer is A. Matters that concern the bedrooms of the nation are private matters. Trudeau sought to reduce government control in such matters. Answers (B) and (C) say the opposite, namely that government control in public matters

Art is visible. However, everything one sees is filtered through certain conditions, some of them historical, and others, natural. The historical conditions include the material which is used oil, colors, and the canvas; second, a certain style, i.e., a system of rules by which things visible are submitted a priori. There can be a general style, for example, the style of Impressionism, or a particular style, for example, the individual ways in which two painters, both impressionists, paint. The natural conditions include certain unchanging psychological laws of sight, for instance, the effects of colors or optical illusions. The conditions of art are nothing but a particular way of interpreting reality. To understand this, one can examine the difference between the classical Greek and the classical Egyptian styles. For the Greeks, the reality of the visible was given by the perspective and the situation in which the object appears; for that reason they presented a person in his individual movements. For the Egyptians, however, this was only the appearance of a transitory moment, which, according to their beliefs, was not real. Therefore, the Egyptians searched for the permanent essence and the typical character in their depiction of an object. For the Egyptians, Greek art was an illusion; for the Greeks, on the other hand, Egyptian art was unrealistic constructivism.

16

http://tailieuso.com/

The way in which reality appears in art must not be regarded on its own. It is affected by many other systems of recognizing reality, including the political, religious, economic, intellectual, and social in short, all the phenomena of human life. Moreover, art is always of a certain epoch, with its particular conception of reality. Thus, when discussing, for example, the art of ancient myth, of medieval Christianity, or that of the technological age, one must be aware that myth, Christianity, or technology was the most salient feature of the epoch. It is paradoxical to understand art as some kind of copy of the fields of experience connected with it. So, for example, it is meaningless for the work of art as such if one compares the landscape of a painting with the landscape, which served the artist as his model. Even if the artist had tried to make what he painted as similar as possible to the model he used, the landscape which he saw is only the matter from which something completely different emerges since he has submitted its view to the a priori conditions of art: namely to the material used (colors, canvas, etc.), to his style, and even to the fact that he paints on a flat surface. Thus one must contemplate a work of art by itself. Even if it is connected to other fields of experience it nevertheless displays something unique which appears in that piece of art and there alone. 1. According to the passage, classical Egyptians did not present a person in his individual movements (A) Because the Greeks believed that the reality of the visible was given by the perspective and the situation in which the object appears (B) Because the Greeks did not believe that the reality of the visible was given by the perspective and the situation in which the object appears (C) Because the Egyptians believed that the reality of the visible was given by the perspective and the situation in which the object appears (D) Because the Egyptians did not believe that the reality of the visible was given by the perspective and the situation in which the object appears (E) Because for the Egyptians, Greek art was an illusion. The best answer is D. The Egyptians believed that the reality of the visible was not given by the perspective and the situation in which the object appears. They thought that this was the appearance of a transitory moment, which was not real. 2. The author mentions which of the following as one of the conditions through which art is seen? (A) Impressionism (B) Optical illusions (C) Nature (D) Perspective (E) Illusions

17

http://tailieuso.com/

The best answer is B. The author states in the first paragraph that art is seen through natural conditions including optical illusions. 3. By asserting that art is filtered through certain conditions (line???), the author suggests which of the following? (A) Even if the artist tried to make what he painted as similar as possible to the model he used, he would not succeed (B) Even if the artist tried to make what he painted as similar as possible to the model he used, it would be impossible to critique it (C) Even if the artist tried to make what he painted as similar as possible to the model he used, the landscape would be affected by many other systems of recognizing reality (D) The way in which reality appears in art influences other systems of recognizing reality, including the political, religious, economic, intellectual, and social (E) The way in which reality appears in art is influenced by other systems of recognizing reality, including the political, religious, economic, intellectual, and social The best answer is E. The author states in paragraph three that the way in which reality appears in art is affected by many other systems of recognizing reality, including the political, religious, economic, intellectual, and social in short, all the phenomena of human life. 4. The author cites the example of psychological laws of sight, for instance, the effects of colors or optical illusions in order to illustrate (A) Laws of nature, which are not affected by history (B) Laws of nature, which are affected by history (C) Laws of nature, which change history (D) Laws of history, which are affected by nature (E) Laws of history, which are not affected by nature The best answer is A. The author details two kinds of conditions, historical and natural. The natural conditions are said to be unchanging, therefore not affected by history.

For a generation of suppressed, restless, working-class youths living in 1960 Jamaica, ska was a medium through which they could find expression. Since its original appearance, ska has resurfaced twice, each time presenting itself in a different guise to a new generation of music aficionados. Overcoming its humble beginnings, it has become one of the twentieth centurys most enduring and influential styles of music.

18

http://tailieuso.com/

Since the early 1940's, Jamaica had adopted and adapted many forms of American musical styles. The predominantly black inhabitants of Jamaica took a liking to rhythm and blues music, importing a considerable number of American records that were showcased at dance halls in the early 1960s. Jamaican musicians took up the elements of rhythm and blues and combined it with traditional Jamaican mento music. The result was the first wave of ska. Musically, ska is a shuffle rhythm similar to mento but with even closer ties to rhythm and blues, placing the accent on the second and fourth beats, often moving in a 12-bar blues frame. The after beat, played on the piano or strummed by a rhythm guitar, came to be characteristic of the form. A horn section, usually consisting of trumpets, trombones, and saxophones, was a vital element. Classic bands, such as the Wailers wrote songs written about Trench Town (a ghetto), rude boys (street thugs), romance, and even religious themes. In 1965, ska began to take a backseat to a newly evolved type of music, called rock steady, which was more dependent than ska had been on rhythm provided by the bass guitar and drums. Ska was later exported by traveling Jamaican artists to Great Britain, where it became known as "blue beat." By the mid 1970's, early British punk bands were infusing reggae, a style of music that came from rock steady, into their music. Near the end of the decade, however, there was a resurgence of the influence of ska because of its upbeat, danceable rhythm. This faster paced ska came to be known as two tone. One of the essential messages of two-tone ska was the promotion of racial harmony and of having fun in the face of subjugation. The third wave of ska began in America around 1990. Bands influenced by the two-tone ska scene began to use punk and metal music to a greater extent. The combination, which is much faster than two tone, sounds very different from the original Jamaican brand of ska. In its three different waves, ska has given voice to seemingly voiceless, downtrodden generations. Each time it resurfaces, a new message is taken up, however, the old messages are never forgotten. 1. The primary purpose of this passage is to Contrast the musical rhythm of two-tone music with original Jamaican ska from which it developed Illustrate various ways in which rhythm and blues has influenced ska music Outline the influences on the various forms of ska music from its inception in Jamaica in early 1960s through its third wave in the 1990s Describe events leading to the inception, rise and final demise of Jamaican ska music

19

http://tailieuso.com/

Trace to evolution of ska music from its inception in Jamaica in early 1960s through its third wave in the 1990s The best answer is E. The passage follows the development of ska. (D) is incorrect because the passage does not deal with the demise, or death, of ska music. 2. According to the passage, Ska music has Been influenced by rhythm and blues, mento and blue beat Been influenced by rhythm and blues, and has influenced mento and rock steady Been influenced by rhythm and blues and rock steady and has influenced punk and metal musicians Influenced rhythm and blues, reggae and metal musicians. Been influenced by mento music and has influenced punk and metal musicians. The best answer is E. Ska music has been influenced, among other things, by mento music. In paragraph five, it is mentioned that ska musicians have influenced both punk and metal musicians. 3. Which of the following statements about ska music is supported by information in the passage? Rock steady is more dependent than ska on the rhythm provided by the bass guitar and drums. Reggae, which counts ska as one of its primary influences, developed only after it was exported by traveling Jamaican artists to Great Britain Skas appeal over the last half century has been limited to voiceless, downtrodden generations. Two-tone is a faster paced form of ska that developed in the late 1970s Mento music places the accent on the second and fourth beats, often moving in a 12-bar blues-frame. The best answer is D. In paragraph four, it says that this faster paced ska came to be known as two tone. 4. The passage suggests that two tone music I. Resurged near the end of the 1970s II. Influenced bands in America in the 1990s III. Promoted of racial harmony and of having fun in the face of oppression. I only II only III only
20

http://tailieuso.com/

II and III only I, and II, and III The best answer is E. (E) is the best choice because all of the facts presented above are supported by the passage.

Coca-Cola, which sold 10 billion cases of soft drinks in 1992, now finds itself asking, where will sales of the next 10 billion cases come from? The answer lies overseas, where income levels and appetites for Western products are at an all time high. Often, the company that gets into a foreign market earliest dominates that country's market. Coke patriarch Robert Woodruff realized this and unleashed a brilliant ploy to make Coke the early bird in many of the major foreign markets. At the height of World War II, Woodruff proclaimed, Wherever American boys were fighting, they'd be able to get a Coke. By the time Pepsi tried to make its first international pitch in the 1950s, Coke had established its brand name along with a powerful distribution network. During the last 40 years, many new markets have emerged. In order to tap into these opportunities, both coke and Pepsi have attempted to find ways to cut through the red tape that thwarts their efforts to conduct business in these new regions. One key maneuver in the soda wars occurred in 1972, when Pepsi signed an agreement with the Soviet Union that made it the first Western product to be sold to consumers in Russia. This landmark agreement gave Pepsi the upper hand. At present, Pepsi has 23 plants in the former Soviet Union and is the leader in the soft-drink industry in Russia. It outsells Coca-Cola by a ratio of 6 to 1 and is seen there as a local brand, similar to Cokes homegrown reputation in Japan. However, Pepsi has also encountered some obstacles. An expected increase in brand loyalty for Pepsi subsequent to its advertising blitz in Russia has not materialized, even though Pepsi produced commercials tailored to the Russian market and sponsored televised concerts. Some analysts believe that Pepsis domination of the Russian market has more to do with pricing. While Pepsi sells for 250 Rubles (about 25 cents) a bottle, Coca-Cola sells for 450 Rubles. Likewise, Pepsi sells their 2 liter economy bottle for 1,300 Rubles, while Coca-Colas 1.5 liters is marketed at 1,800 rubles. On the other hand, Coca-Cola only made its first inroads into Russia 2 years ago. What's more, although Coca-Cola's bottle and label give it a highclass image, Russians do not perceive Coca-Cola as a premium brand in the Russian market. Consequently, it has so far been unable to capture a market share.
21

http://tailieuso.com/

1. According to the passage, all of the following have been used to attract customers to buy a one of the two brands of soft drink mentioned in the passage EXCEPT Offering soft drinks for a limited time at specially reduced prices Sponsoring televised concerts Designing a bottle and label to create a high-class image Staging an advertising blitz including commercials tailored to the local market Being the first country to enter a foreign market The best answer is A. Answers (b), (C), (D) and (E) are all mentioned in the passage as ploys used by either Pepsi or Coca Cola to attract new customers 2. The passage suggests which of the following about the Russian soft drink market? Price is an unimportant factor in the Russian soft drink market Two liter economy bottles are more marketable than 1.5-liter economy bottles, especially those sporting a high-class image. One and a half liter economy bottles are more marketable than two liter economy bottles, if sold at a lower price. Russian consumers are more likely to purchase a product if the perceive it to be a local brand The Russian soft drink market is saturated with local brands. The best answer is D. The passage states that one of the factors contributing to Pepsis success in Russia is its perception by the public as a local brand. 3. The primary purpose of the passage is to Review the marketing history of two soft drink giants Contrast two different approaches to marketing soft drinks in the global market Refute the traditional explanation for Pepsis success in the Russian soft drink market Compare how well two soft drink companies have succeeded in a new foreign market Explain why two soft drink companies have succeeded in a new foreign market The best answer is D. The passage mainly compares Pepsis success in a new foreign market, Russia, with Coca Colas relative failure. (A) is too general. (B) is incorrect because both companies have the same general approach. 4. Which of the following best describes the relation of the first paragraph to the passage as a whole?

22

http://tailieuso.com/

It poses a question to be answered It outlines an objective whose attainment will be discussed It outlines a process to be analyzed It advances and argument to be disputed It introduces conflicting arguments to be reconciled The best answer is B. The first paragraph asks and answers the question of where the sale of the next 10 billion cases will come from, namely foreign markets. The rest of the passage discusses ways in which the objective of conquering foreign markets is accomplished.

With the proliferation of electronic technologies in the latter part of the twentieth century, many aspects of cultural practice have been redefined. The eradication of physical boundaries that limit discourse and information access has had profound effects upon the manner in which we conduct democracy. Yet, opinions strongly differ over whether or not the growth of electronic networks will result in expanded democracy. On one side of the debate are antiutopians who fear that with the intrusion of the Internet into many facets of life, personal freedom will be impeded and the existing rift between the "haves" and "have-nots" in society will grow. On the other side, many 'cyber-utopians' believe that new technologies can eliminate the democracy of elected representatives with which so many people are dissatisfied. The Internet, they say, will allow for a true participatory democracy in which citizens can govern themselves without the interference of bureaucrats and legislators. Neither of these theories by themselves can fully address the role of democracy in the age of information. As debates about censorship and encryption have shown, government regulation of the Internet can result in violations of the basic rights of speech set forth in the constitution of the United States. Yet, groups that preach Big Brother theories of paranoia tend to neglect the fact that new technologies can help balance the injustices of traditional power found in a centralized government. At the same time, the likelihood of doing away with the present system of democracy in favor of complete and pure selfgovernance seems impossible, and likely undesirable. Both arguments about the future of the way in which discourse will occur highlight the inherent relationship between communication and democracy. Perhaps a more useful model for the study of this dynamic can be found in the model of the public sphere proposed by Jorgen Habermas. In this realm, free and diverse equals come together to deliberate and discuss pertinent issues without the impediment of external coercion. The ensuing dialogue transpires in a profoundly democratic forum. The dispensing of traditional hierarchies that occurs on the Internet appears to make possible the type of categories necessary for Habermas ideal speech situation to occur.
23

http://tailieuso.com/

However, postmodern critics indicate that the autonomous individual no longer exists in a world where our identities are constructed as much for us as by us. And indeed, much of the postmodern notion of self seems to fit closely with reconfigurations of the subject brought on by electronic technologies. The question that arises then is, how might the reconfiguration of communication enabled by the Internet work to create a new form of 'cyber-democracy that better represents citizens' interests? 1. According to the passage, the 'cyber-utopians' mentioned in the passage would most likely be in favor of which of the following innovations? Every new legislation would be voted by every registered voter on the Internet Government would increase the regulation of the Internet to include a curtailing of politically biased messages Government would decrease the regulation of the Internet including regulation of politically biased messages Discourse in legislative assemblies would be broadcast over the Internet New technologies would gradually replace all forms of democracy The best answer is A. According to the passage, 'cyber-utopians' believe that through using the Internet, there could be a true participatory democracy, meaning that all citizens, not just elected representatives could make legislative decisions. 2. The passage supports which of the following statements about government regulation of the Internet? Government regulation of the Internet can result in infringements upon citizens constitutional rights of free speech Government regulation of the Internet can ensure against infringements upon citizens constitutional rights of free speech Government regulation of the Internet will make pure self-governance possible Government regulation of the Internet will promote new technologies that can help balance the injustices of traditional power Government regulation of the Internet will eradicate physical boundaries that limit discourse and information The best answer is A. As stated in paragraph three, government regulation of the Internet can result in violations of the basic rights of speech set forth in the constitution.

3. The author is primarily concerned with Advocating the use of the electronic technologies to improve democracy Challenging the assumptions on which a theory of modern democracy is based
24

http://tailieuso.com/

Describing events leading to the discovery of democratic uses of electronic technologies Explaining the importance of electronic technologies to modern politics Examining the relationship between Internet communication and democracy The best answer is E. The answer is not (A) because the author does not reach any conclusions. (D) is incorrect because it does not discuss modern politics in general. 4. According to the passage, which of the following is considered by postmodern critics to be a threat to the notion of self? The interference of bureaucrats and legislators. The proliferation of electronic technologies. Reconfigurations of the subject brought on by electronic technologies. Traditional hierarchies that occur on the Internet. The impediment of external coercion. The best answer is C. In the last paragraph, it says that much of the postmodern notion of self seems to fit closely with reconfigurations of the subject brought on by electronic technologies.

Men are primarily and secondarily socialized into believing certain characteristics are definitive in determining their masculinity. These characteristics range from playing violently to not crying when they are injured. The socialization of masculinity in our society begins as early as the first stages of infancy, with awareness of adult gender role differences being internalized by children as young as two years old. Studies show that advertising imagery equates masculinity with violence by portraying the trait of aggression as instrumental to establishing their masculinity. Lee Bowker, who researched the influence of advertisements on youth, asserts that toy advertisements featuring only boys depict aggressive behavior and that the aggressive behavior produces positive consequences more often than negative. Bowker also looked at commercials with boys that contain references to domination. His results indicated that 68.6% of the commercials positioned toward boys contain incidents of verbal and physical aggression. However there were no cross gender displays of aggressive behavior. Interestingly, not one single-sex commercial featuring girls showed any act of aggression. Bowkers research helps explain that it is not just the reinforcement of a childs

25

http://tailieuso.com/

close caretakers that lends legitimacy to aggressive masculine tendencies but society as a whole, using the medium of television. William Pollack, a Harvard clinical psychologist, talks about how males have been put in a "gender straightjacket" that leads to anger, despair and often violence. Pollack states that society asks men to put a whole range of feelings and emotions behind a mask and shames them if they display any emotion. Pollack contends that boys are shame phobics, even killing, in extreme cases, to avoid dishonor. It appears that the standard defined by society allows men to express their emotion only through anger. Ironically, though these rigid stereotypes of what it means to be a man have been inculcated from an early age, men are often criticized for being one-dimensional in their behavior and emotions. Women often verbalize a desire for males to be sensitive and express their emotions. But male insensitivity is the culmination of a societal indoctrination begun at birth. Realistically, men are in a damned if they do, damned if they dont situation. If they fail to show their emotions, they are berated for being detached from the essence of what constitutes a human being. On the other hand, if a male decides to expose his emotions, he is often branded effeminate and regarded as inferior to other males who stick closer to their genders traditional doctrine. 1. According to the passage, the television commercials examined by Bowker Showed boys in more acts of verbal and physical aggression than of domination Showed boys in more acts of domination than of verbal and physical aggression Showed boys in acts of verbal and physical aggression only towards other boys Showed boys in acts of verbal and physical aggression only towards other girls Showed boys in acts of verbal and physical aggression towards other boys and girls The best answer is C. Bowkers research did not find any cross gender displays of aggressive behavior, i.e. aggression of one gender to another 2. According to Pollack, one of the reasons for male violence is that Society shames men who display feelings and emotions other than anger Men kill in extreme cases to avoid dishonor Men are often criticized for being one-dimensional in their behavior and emotions Society uses television as a symbol of its desires Reinforcement from childs close caretakers lends legitimacy to aggressive masculine behavior

26

http://tailieuso.com/

The best answer is A. (B) is incorrect because it does not give a reason for violence. (C) is a result of the conditioning that leads to violence, not a reason. (D) and (E) are incorrect because they are not opinions expressed by Pollack. 3. The passage suggests that, when compared with television advertisement featuring boys, advertisements that had only girls were found To have more references to domination To be 68.6% less aggressive To be remarkably similar in focus and content To be replete with extensive examples of cross gender aggression To be void of any acts of aggression The best answer is E. Bowker found that not one single-sex commercial featuring girls showed any act of aggression. 4. Pollack uses the term gender straightjacket to emphasize The narrow range of emotion that society allows men to express The broad range of emotion that society allows men to express The danger of anger, despair and violence towards men The danger of anger, despair and violence perpetrated by men The wide range of feelings that men actually experience The best answer is A. Society does not allow men to act in ways it has deemed inappropriate, hence Pollack considers the male gender to be in a straightjacket.

27

http://tailieuso.com/

Juror anonymity was unknown to American common law and jurisprudence in the countrys first two centuries. Anonymity was first employed in federal prosecutions of organized crime in New York in the 1980's. Although anonymous juries are unusual since they are typically only empanelled in organized-crime cases, its use has spread more recently to widely publicized cases, such as the federal prosecution of police officers accused of beating Rodney King and the trial of those accused of the 1993 World Trade Center bombing. In these cases, attorneys selected a jury from a panel of prospective jurors whose names, addresses, ethnic backgrounds and religious affiliations remained unknown to either side. This unorthodox procedure, designed to protect jurors from outside influence and the fear of retaliation, has occasionally been employed in New York federal courts since the trial of drug kingpin Leroy "Nicky" Barnes. Despite apparent benefits, critics assail anonymous juries on the grounds that they are an infringement of the sixth amendment guarantee of an impartial jury and because they present a serious and unnecessary erosion of the presumption of innocence. Since many attorneys believe trials are frequently won or lost during jury selection, any procedure diminishing the role of counsel in the procedure necessitates close scrutiny and criticism. Opponents of anonymous juries argue that the procedure restricts meaningful voir dire, (questioning of the jury panel), and thereby undermines the defendant's sixth amendment right to an impartial jury. Critics also claim that jurors interpret their anonymity as proof of the defendant's criminal proclivity, thereby subverting the presumption of innocence. However, consistent with due process and the sixth amendment, the trial judge may refuse to ask prospective jurors any questions not reasonably calculated to expose biases or prejudices relevant to the case. Although addresses and group affiliations may indicate significant potential for bias, attorneys do not have an unfettered right to this information in every circumstance. Denying access to these facts may indeed constrain an attorney's ability to assemble an ideal jury, but it violates no constitutional right.

1. The primary purpose of the passage is to Enumerate reasons why anonymous juries are unconstitutional Discuss whether anonymous juries are an infringement of the sixth amendment Identify a shortcoming in a scholarly approach to jurisprudence Define the concept of anonymous juries and explore efforts taken over the last twenty years to increase their use Review strategies for ensuring that anonymous juries will not infringe on the constitutional right to a fair trial of ones peers
28

http://tailieuso.com/

The best answer is B. The passage introduces the concept of anonymous juries and goes on to discuss their constitutionality. 2. It can be inferred from the passage that a jurors ethnic background and religious affiliation Is considered by defendants not to have a significant effect on the outcome of their trials Is considered by defendants to have a significant effect on the outcome of their trials Would be unlikely to have a significant effect on the verdict of a trial Is considered by attorneys likely to have a significant effect on the verdict of a trial Is considered by attorneys unlikely to have a significant effect on the verdict of a trial in a widely publicized case The best answer is D. In paragraph three it states that many attorneys believe trials are frequently won or lost during jury selection. The passage gives no information on what defendant think about anonymous juries. 3. One function of the fourth paragraph of the passage is to Qualify the extent to which a previously introduced viewpoint may be relevant Expose the flaw in a criticism put forth in a previous paragraph Introduce information that supports a theory put forth in a previous paragraph Support an argument in favor of a given interpretation of a situation Show the chain of reasoning that led to the conclusions of a specific study The best answer is B. Critics of anonymous juries base their arguments on the fact that these juries are unconstitutional. In the fourth paragraph, the author explains that while anonymous juries may not be ideal, they are not unconstitutional.

4. Which of the following, if true, would ensure that anonymous juries are not an erosion of the presumption of innocence? Anonymous juries are used in all court cases, regardless of identity of the defendant. Anonymous juries are used in all court cases involving previously convicted defendants. Anonymous juries are used in all court cases, involving never before convicted defendants. Anonymous juries are used in all widely publicized court cases, regardless of identity of the defendant.
29

http://tailieuso.com/

Anonymous juries are used in all widely publicized court cases, involving previously convicted defendants. The best answer is A. Anonymous juries are a potential erosion of the presumption of innocence only because they are now used in cases in which there is a perceived danger to the jurist, which presupposes a defendant capable of perpetrating a crime. If anonymous juries were used in each and every case, they would no longer differentiate between dangerous and harmless defendants.

Alexander Calder was one of the most innovative and original American artists of the twentieth century. Calder arrived in Paris in 1926 and devoted himself to a innovative project comprised of animals made out of wire, scraps of cloth, wood, cork, labels, bits of scrap metal and pieces of rubber that he called the Circus. During his performances, Calder invented ways to simulate the flight of birds: These are little bits of white paper, with a hole and slight weight on each one, which flutter down several variously coiled thin steel wires which I jiggle so that they flutter down like doves. The Circus was the laboratory of Calders work; in it he experimented with new formulas and techniques. By 1930, Calder's Circus had developed into one of the real successes of the Montparnasse art world attracting the attention of such renowned artists as Fernand Leger and Joan Miro. Encouragement from the upper echelons of the Parisian art scene undoubtedly led him to try more serious experiments in wire sculptures. Calder eventually becoming interested in the movement of objects, some of which he motorized. In 1933, Calder completed Object with Red Discs, a sculpture he described as a two-meter rod with a heavy sphere, suspended from the apex of a wire, giving it a cantilever effect. It had five thin aluminum discs projected at right angels from five wires, held in position by a spherical counterweight. With this new creation, the idea of the mobile was born. In creating a work named Constellations in 1943, Calder explored the plastic possibilities of mobiles; he used small pieces of wood, which he shaped and sometimes painted. From this point on, Calders ambition changed focus. He sought more challenging designs. One of Calders objectives was to display objects in the air, giving the viewer the experience of finding new skies filled with moving and colored constellations. Calder accomplished this in Acoustic Ceiling (1954). Calders humor was evident in such works as Le Bougnat (1959) and The Pagoda (1963). Later, Calder cut fantastic animals from sheet metal, creating La Vache and Elephant (both 1970) and a mobile entitled Nervous Wreck (1976), which represents the red skeleton of a fish.

30

http://tailieuso.com/

Calder defined volume without mass and incorporated movement and time in art. His inventions, which redefined certain basic principles of sculpture, have established him as the most innovative sculptor of the twentieth century. 1. According to the passage, which of the following is an accurate statement about Object with Red Discs? It was the first mobile created by Calder. It was one of the many mobiles without motors created by Calder. It was one of the many motorized mobiles created by Calder. It was the first motorized mobile created by Calder. It was the first of the many mobiles without motors created by Calder. The best answer is A. According to the passage, Object with Red Discs is Calders first mobile. It states that Calder became interested in the movement of objects, some of which he motorized, but there is no information given on whether this particular sculpture was motorized. 2. According to the passage, all of the following are characteristic of Calders work EXCEPT Calder was known to infuse humor into some of his creation Calder suspended objects from each other Calder motorized some of his creations Calder used materials such as metal, cloth, wood, rubber, cork Calder suspended glass from thin metal wires to create a cantilever effect The best answer is E. The passage makes no mention of glass as one of the materials Calder used. 3. The authors attitude toward the mobiles of Alexander Calder is best described as Hesitance Detachment Amusement Admiration Indifference The best answer is D. The author presents only a positive criticism of Calder, stating that he is the most innovative sculptor of the twentieth century. 4. It can be inferred from the passage that which of the following statement was true of the Parisian art scene?

31

http://tailieuso.com/

The work of Fernand Leger and Joan Miro was influenced by that of Alexander Calder. The work of Alexander Calder was influenced by that of Fernand Leger and Joan Miro. Fernand Leger and Joan Miro had earned success in the art world before Alexander Calder. Alexander Calder had earned success in the art world before Fernand Leger and Joan Miro. Calders Circus earned more accolades from the upper echelons of the Parisian art scene than any other work in its time. The best answer is C. According to the passage, Calders early work attracting the attention of such renowned artists as Fernand Leger and Joan Miro. It can be inferred that Leger and Miro were already famous when Calder was just starting out.

Intuitively, intellectual skills and perceptual-motor skills seem very different because perceptual-motor skills appear more primitive. Ontogenetically, perceptual-motor skills develop before intellectual skills, or at least before most intellectual skills are manifested. Phylogenetically, creatures "high on the evolutionary ladder" are more obviously capable of intellectual skills than are creatures "lower down ". Perceptual-motor skills also seem more closely tied to specific forms of expression. Being a chess player does not mean one can only play with pieces of a certain size, that one can only move pieces with one's right hand, and so on. By contrast, being a violinist means one can play an instrument whose size occupies a fairly narrow range and that one must play with a rather rigid assignment of functions to effectors (bowing with the right hand, and fingering with the left). The seeming narrowness of this perceptual-motor skill expression, contrasted with the seeming openness of intellectual skill expression, seems to follow from intellectual skills having symbolic outcomes and perceptual-motor skills having non-symbolic outcomes. Symbolic outcomes need not be realized in specific ways and can rely on abstract rules. Non-symbolic outcomes, by contrast, need more specific forms of realization and seem to depend on restricted associations between stimuli and responses .

32

http://tailieuso.com/

Another difference between intellectual and perceptual-motor skills is that the two kinds of skill seem to be represented in different parts of the brain. For example, structures homologous to the optic tectum, a nucleus located on the dorsal surface of the midbrain, have a common function in all vertebrates-coordinating visual, auditory, and somatosensory information relevant to the control of orienting movements of the eyes, ears, and head. Similarities in structure and function between these and other brain areas associated with perceptual-motor behavior suggest that mechanisms for control of perceptualmotor skills are both highly specialized and conserved across species. In contrast, what distinguishes the human brain from the brains of other species -even closely related ones -- is the differential growth of brain regions most strongly associated with intellectual skills, such as the association areas of the cerebral cortex. The contention that these areas serve intellectual functions is supported by a large body of clinical and experimental literature. Together, these diverse sources of information suggest that perceptual-motor and intellectual skills depend on distinct brain circuits . 1. The passage is chiefly concerned with Presenting a new theory and describing a new method to test that theory Suggesting an alternative to an outdated research method Demonstrating that perceptual-motor skills are closely tied to specific forms of expression Arguing that two seemingly dissimilar skills are more alike than was previously assumed Presenting evidence on two dissimilar skills that resolves a contradiction The best answer is D. The passage presents evidence to back the claims that intellectual skills and perceptual-motor skills are more similar than was once believed.

2. The author mentions the game of chess in paragraph two primarily in order to Present an example of an intellectual skill the mastery of which is not closely tied to specific forms of expression. Present an example of an intellectual skill the mastery of which is closely tied to specific forms of expression.

33

http://tailieuso.com/

Present an example of a skill that is both an intellectual skill and a perceptualmotor skill, the mastery of which is closely tied to specific forms of expression. Present an example of a perceptual-motor skill the mastery of which is not closely tied to specific forms of expression. Present an example of a perceptual-motor skill the mastery of which is closely tied to specific forms of expression. The best answer is A. Chess is an intellectual skill the mastery of which is not closely tied to specific forms of expression. In other words, once one has learned to play the game, one can vary the way one plays, by using ones left hand instead of ones right, for instance. 3. It can be inferred from the passage that the optic tectum Functions similarly in animal and in plants Functions similarly in vertebrates and invertebrates Is located in a comparable area of the brains of humans and giraffes Coordinates somatosensory moment in snakes Has a much more sophisticated structure than the cerebral cortex The best answer is C. According to the passage, the optic tectum occupies the same area of the brain in all vertebrates (animals with a spinal column).

4. The passage provides support for which of the following statements Creatures "high on the evolutionary ladder" are not less capable of perceptualmotor skills than are creatures "lower down ". Playing a violin is a perceptual-motor skill. The shape and size of the cerebral cortex is what distinguishes the human brain from the brains of other species Literature is an intellectual activity that is understood using the cerebral cortex of area of the brain. Perceptual-motor and intellectual skills exploit the same brain circuits. The best answer is B. The author uses the violin as an example of a perceptualmotor skill.

Considerable debate exists in the self-perception literature over the impact of positively biased self-perceptions on social and psychological functioning. Positively based self-perceptions are those in which an individual has a more positive opinion of himself than objective indicators warrant. One view suggests that positive perceptual biases are characteristic of normal human thought across a variety of domains and correlate positively with good mental and psychological health. Certain researchers and clinicians have even
34

http://tailieuso.com/

proposed that by boosting self-concepts, symptoms of depression and levels of aggression may be reduced. Investigators on the other side of the debate maintain that when most positive self-perceptions are compared to an objective criterion, they appear neither positively biased nor adaptive. In fact, Baumeister, Smart, and Boden suggest that positively biased self-concepts may have a dark side. They proposed that it is persons with very positive self-views who are prone to be aggressive. As a result, building up individuals' self-perceptions may serve only to increase levels of aggression rather than curb them. According to Baumeister et al., not all individuals with positive selfperceptions are going to be interpersonally aggressive. Rather, individuals who are extremely positive in their perceptions of themselves and their functioning are proposed to be the most likely to become angry and potentially violent. The mechanism that triggers aggressive behavior by these individuals has been suggested to be negative social feedback that challenges their positive selfviews. Such threats to positive self-esteem give rise to anger and hostility. If negative social information is encountered that challenges established positive self-perceptions, Baumeister et al. propose that individuals must choose to either accept the feedback and lower their self-perceptions or reject the feedback to maintain their positive self-views. The chosen reaction then influences their subsequent affective states and behavioral expressions. By accepting the external appraisals and adjusting self-perceptions downward, dysphoric feelings and social withdrawal may result. Conversely, the rejection of the validity of the unfavorable feedback results in feelings of anger and resentment toward the source of the threat. Dodge and colleagues demonstrated that children who interpret social cues as threatening direct their anger and aggression at the peers who gave the negative evaluations. Anger stemming from the receipt of social criticism is a way to deny the legitimacy of the negative information. By directing hostile reactions toward the source of the negative feedback, the influx of disconfirming information may end. Unless individuals react against the self-esteem threat, they may be compelled to revise their self-concepts negatively, in line with the information provided. By discounting the negative social feedback, individuals can protect themselves from dysphoric feelings and maintain their positively biased self-perceptions, but they may be setting themselves up to become interpersonally aggressive. Although positively biased self-perceptions may place individuals at risk for negative social feedback and subsequent increases in aggressive behavior, not all positive self-concepts are suggested to be harmful. The relationship between positive self-perceptions and aggression may depend on the degree of perceptual distortion (i.e., moderate vs. extreme distortion). Baumeister (1989) and Baumeister et al. (1996) proposed that an optimal range of moderate bias might exist within which mental health is encouraged. Maladjustment in
35

http://tailieuso.com/

psychological and social functioning is suggested to occur when the degree of bias of self-perceptions shifts from moderate to extreme levels. Extremely negative and positive perceptual bias would be related to different but equally harmful difficulties. 1. The primary purpose of this passage is to Present two explanations of a phenomenon and reconcile the differences between them Discuss a plan for investigation of a phenomenon that is not yet fully understood Challenge the validity of a theory by presenting evidence that the opposite is true in some cases Summarize two theories and suggest a third theory that overcomes the problems encountered in the first two Present evidence that resolves a contradiction The best answer is C. The passage challenges the validity of the theory that positively biased self-perception is closely correlated with normal human thought and good mental health by showing how a very high self-perception may lead to violent behavior.

2. According to the passage, which of the following is mentioned as a factor in determining whether an individual with positively biased self-perception would actually be likely to perform an act of violence? The gap between what the individual thinks about himself and how good he really is The gender of the individual The anger level of the individuals peers The individuals ability to ignore dysphoric feelings The validity of the unfavorable feedback The best answer is A. It is the distortion between how the person really is and how he perceives himself that is the most important factor, as opposed to the level of self-perception itself.

3. The passage discusses the likelihood of violence stemming from which of the following types of individuals? An individual with a moderately positive self-perception who receives negative feedback from his peers An individual with a highly positive self-perception who receives negative feedback from his peers
36

http://tailieuso.com/

An individual with a highly negative self-perception who receives positive feedback from his peers An individual with a highly negative self-perception who receives negative feedback from his peers An individual with a dark side who receives negative feedback from his peers The best answer is B. According to the passage, when negative social information is encountered by a person with a highly elevated level of selfperception, it may lead to violence.

4. One function of the fifth paragraph is to State a conclusion about facts presented in an earlier paragraph Show the chain of reasoning that led to the conclusions of a specific study Qualify the extent to which a previously presented theory may apply Introduce information that confirms an established theory Provide examples that support a new theory The best answer is C. Up to the fifth paragraph, the passage discusses the possibility that individuals with highly positive self-perception are prone to performing violent acts. Paragraph three qualifies this by stating that not all positive self-concepts are suggested to be harmful.

37

http://tailieuso.com/

Gastrostomy tubes are commonly used to provide nutrition and hydration for patients unwilling or unable to maintain an adequate oral intake. Among hospitalized patients aged 65 years or older in the United States, the number undergoing placement of a gastrostomy tube increased from 61000 in 1988 to 121000 in 1995. In 1990 and 1991, roughly one in every hundred hospitalized patients aged 85 years or older received a gastrostomy tube. The short-term mortality rates following gastrostomy placement are high. In a cohort of more than 7000 American veterans who underwent placement of percutaneous endoscopic gastrostomy tubes between 1990 and 1992, median survival was 7.5 months and 1-year mortality was 59%. Among Medicare beneficiaries receiving gastrostomy tubes in 1991, 30-day and 1-year mortality was 24% and 63%, respectively. Because tube insertion itself is only rarely associated with fatal complications, the high short-term mortality clearly reflects a substantial underlying co-morbidity in this population. Most patients receiving gastrostomy tubes have advanced dementia, other types of severe neurological impairment, cancer, or advanced failure of other internal organs. The growing use of tube feeding in a population with limited life expectancy inevitably raises the following question: Do physicians discuss the benefits and burdens of tube feeding adequately with patients or surrogate decision-makers before gastrostomy tubes are inserted? Assessing benefits and burdens is an integral part of informed decision-making and should precede any elective lifesustaining intervention. However, anecdotal observations and a recent interview study raise serious questions about the quality of the informed consent process preceding the insertion of gastrostomy tubes. A small body of literature suggests that fully informed patients or their surrogates might in fact decline permanent tube feeding at a higher-thanexpected rate. For example, in the study by Callahan et al, nearly half of the patients undergoing gastrostomy placement (or their surrogates) reported that no alternatives had been discussed before insertion of the tube. O'Brien and colleagues asked 379 mentally competent nursing home residents if they would want a gastrostomy tube if they became unable to eat because of permanent brain damage; only 33% expressed a preference for tube feedings in this circumstance. In an interview study of 121 competent patients with amyotrophic lateral sclerosis, only 28% favored feeding by gastrostomy .

1. The passage is primarily concerned with (A) the morbidity and mortality rates associated with the use of gastrostomy tubes
38

http://tailieuso.com/

(B) the proliferation of the use of gastronomy tubes in patients aged 85 years or older (C) whether physicians adequately discuss the benefits and burdens of tube feeding with patients or surrogate decision-makers before gastrostomy tubes are inserted (D) the growing number of patients undergoing placement of a gastrostomy tube (E) the complications that often follow the insertion of a percutaneous endoscopic gastrostomy tube The best answer is C. The question raised in the passage is not whether to use gastrostomy tubes and what the dangers of using one might be, rather the passage examines the issue of informed consent before the insertion of a tube. 2. According to the passage, the high mortality rate following gastrostomy found in research cited in paragraph two is not necessarily a direct result of the placement of a gastrostomy tube because (A) tube insertion itself is only rarely associated with fatal complications (B) 30-day and 1-year mortality was 24% and 63% respectively in 1991 among Medicare beneficiaries (C) only cases in which the percutaneous endoscopic gastrostomy tube was incorrectly place were examined (D) all the patients in the studies mentioned suffered from life threatening diseases (E) most of the patients in the studies mentioned suffered from life threatening diseases The best answer is E. In paragraph two it is stated that most subject tested suffered from one of a number of potentially fatal diseases. Hence, the mortality rate found in patients that receiving a tube, was not necessarily due to the use of the tube, but to the disease itself or to some other treatment. 3. It can be inferred from the passage that (A) there are alternatives to the use of gastrostomy tubes to provide nutrition but not hydration for patients unwilling or unable to maintain an adequate oral intake. (B) there are alternatives to the use of gastrostomy tubes to provide hydration but not nutrition for patients unwilling or unable to maintain an adequate oral intake. (C) there are alternatives to the use of gastrostomy tubes to provide nutrition and hydration for patients unwilling or unable to maintain an adequate oral intake. (D) the alternatives to gastrostomy tubes do not provide adequate nutrition and hydration (E) the alternatives to gastrostomy tubes provide more adequate nutrition and hydration than the gastrostomy tubes
39

http://tailieuso.com/

The best answer is C. The passage suggests that if patients were offered alternatives to gastrostomy tubes, some of them would choose them. We can infer from this that alternatives exist. 4. The author uses the word only in line ??? [third line from the end] most likely in order to (A) highlight the oddity of the decision of the patients (B) emphasize the relatively low percentage of patients that would opt for a gastrostomy tube if given the choice (C) point out the limited value of inserting a gastrostomy tube (D) distinguish the primary factor in the decision making process of brain damaged patients (E) single out a unique merit of gastrostomy tubes for brain damaged patients The best answer is B. The passage draws attention to the fact that patients and their surrogates are often not asked if they agree to the use of a gastrostomy tube. O'Briens study indicates that a relatively small percentage only 33% stated that they would want the tube if they were unable to eat because of permanent brain damage.

40

http://tailieuso.com/

A ragtag group of idealistic hackers scattered round the world has created software and devised a revolutionary method for writing it that poses a direct threat to Microsoft's revenue. Their programs are already running most of the Internet. According to a survey by the British consultancy Netcraft, the Web server software Apache is used by more than half of all websites. Furthermore, software named Sendmail moves nearly every e-mail message across the Internet, while the BIND program acts as a traffic cop for most of the global network, directing messages down the right connections to their final destinations The proven robustness of these programs is worry enough for Microsoft. To make matters worse, all of them, and many others, are completely free in two senses: one does not have to pay for them, and the "source code" in which they are written is openly available. Additionally, one may modify the programs and even sell the result. The "open source" movement is Microsoft's worst nightmare: a group of programmers that it cannot out-compete because its members are not motivated by profit, and which it cannot buy because it does not exist as a formal company. In the vanguard of the open source movement is Linux, started in 1991 by a 21year-old Finn, Linus Torvalds, who wanted to write a free alternative to Unix, a popular but costly operating system. Today Linux is used by an estimated 7 million people, and the number is growing rapidly. One of Linux's advantages is that it runs on almost any hardware, from multi-processor supercomputers down to Palm Pilots. It is compact (it can fit on a floppy), highly efficient and very fast . Torvalds did not invent the idea of software that is doubly free but he has stumbled upon and developed a crucially important Darwinian dynamic. In a commercial software company, every program is carefully planned, and writing tasks are allotted unilaterally by the project leader. Linux is different. It is designed as a series of modules, and anyone can work on any of these interlocking elements. Whether ones work gets included in the final release depends on the consensus view of how good it is--natural selection in action. The only reward anyone, even Torvalds, gets for this work is kudos from fellow hackers. That is enough, it seems, to attract a flow of keen recruits, typically computer science students or software engineers who code Linux on the side. Such purposive anarchy is made possible by the Internet. Trial versions of programs can be downloaded, and comments sent back to the authors, wherever they are. Programs frequently evolve on a daily basis. With the help of the Internet, the Linux model exploits the ingenuity of hundreds of
41

http://tailieuso.com/

programmers and hundreds of thousands of testers. It is a pool of creativity that Microsoft, with its huge resources, will never be able to match. 1. The author mentions that a particular bit of software is included into the final release of Linux only by consensus as an example of (A) how writing tasks are allotted unilaterally by the project leader (B) the modularity of Linux (C) Linuxs unorthodox system of rewards (D) the careful planning that goes into Linux software (E) the process of natural selection at work in the field of computer programming The best answer is E. The author describes the workings of Linux as having a Darwinian dynamic it works on the principle of natural selection. Only the fittest software is used. 2. It can be inferred from the passage that if Linux existed as a formal company (A) it would be used by more than the estimated 7 million people that currently use it (B) its software would be more costly than Unix (C) its software would still be less costly than Unix (D) Microsoft would try to acquire it (E) it could compete with Microsoft The best answer is D. According to the passage, Microsoft cannot buy Linux because it does not exist as a formal company.

3. According to the passage, programmers are motivated to create software for Linux I. to earn accolades II. for pecuniary gain III. to learn how to successfully hack software (A) I only (B) II only (C) I and II only (D) II and III only (E) I, II and III The best answer is A. The author states that the only reward anyone gets for work on Linux is kudos, or accolades, from fellow hackers. 4. The passage mentions each of the following as factors contributing to the success of groups creating open source software EXCEPT (A) the groups use the Internet to gather together hundreds of thousands of testers (B) the groups do not exist as formal companies
42

http://tailieuso.com/

(C) the products are designed to work on Palm Pilots (D) the groups are not motivated by profit (E) the programs evolve frequently The best answer is C. The author mentions Palm Pilots only in relation to the Linux program. No information is given on whether other open source software is versatile enough to run on Palm Pilots.

The cyclic rise and fall in population size that has been observed in animal and pre-modern human populations reflects sequential phases of population growth and decline. Disregarding the possible selective influences of migration, these cyclic movements must ultimately be accounted for in terms of the range of variation in fertility and mortality. No downward trend in mortality is apparent in any country before the middle of the eighteenth century, about the same time that population growth began to demonstrate an exponential curve. The initial period of sustained population growth in nearly every country for which reliable data are available corresponds with at least two decisive changes in the death rate. First, the fluctuations in mortality became less frequent and less drastic. Second, the initial, slow--sometimes imperceptible--decline in mortality gradually gained momentum and eventually stabilized at relatively low levels in the twentieth century. Thus, steady increases in life expectancy, progressively diminishing death rates and more stable and predictable mortality patterns have accompanied the persistent increments in world population. In England and Wales, Japan, Ceylon and Chile an exponential pattern of population growth has accompanied the downward trend in mortality. In England and Wales, where the transition from high to low vital rates occurred over two centuries, the exponential growth curve was attenuated only after fertility fell and approached the low level of mortality; this pattern was less apparent for Japan, where an accelerated transition occurred over several decades. Although data concerning the relative effects of mortality and fertility on population growth are incomplete for the early transitional period, it seems likely that a significant though temporary increase in fertility may have added momentum to the population explosion set off by steady improvements in survivorship. The influence of fertility is particularly apparent in the rapid population growth of currently developing nations that have not yet completed their transitions, for example, Chile and Ceylon. In most of these developing countries, the death rate has declined rapidly in recent years, especially since World War II, and the birth rate has remained high with minor fluctuations. This sudden widening of the demographic gap has produced unprecedented high rates of population growth. 1. Which of the following best describes the content of the passage?
43

http://tailieuso.com/

(A) A discussion of how recently developed methods of monitoring population growth differ from older methods (B) A description of some of the factors effecting cyclic changes in population (C) A chronology of the development of different methods for monitoring population growth (D) A proposal for improving the accuracy of current methods to monitor fluctuations in population (E) An argument concerning the nature of the exponential pattern of population growth has accompanied the downward trend in mortality The best answer is B. (A) is incorrect because no recently developed methods are mentioned in the passage. (C) is wrong because the information given is not chronological. (D) and (E) are incorrect because the passage neither proposes nor argues anything. 2. According to the passage, the fertility rate (A) in Japan has remained constant over the last several decades (B) in England and Wales has remained constant over the last two centuries (C) fell at a similar rate in England, Wales and Japan (D) fell at a faster rate in Japan than in England and Wales (E) fell at a slower rate in Japan than in England and Wales The best answer is D. According to the passage the transition from a high fertility rate to a low one occurred over two centuries in England and Wales. The fertility fell in Japan over a period of several decades. 3. According to the passage, population increase in currently developing nations is a result of (A) the recent decline in death rate, especially over the last 50 years, and a birth rate that is consistently high with minor fluctuations. (B) the recent decline in birth rate, especially over the last 50 years, and a death rate that is consistently high with minor fluctuations. (C) the recent decline in death rate, especially over the last 150 years, and a birth rate that is consistently high with minor fluctuations. (D) the recent decline in birth rate, especially over the last 150 years, and a death rate that is consistently high with minor fluctuations. (E) the recent rise in death rate, especially over the last 50 years, accompanied by a similar rise in birth rate. The best answer is A. According to the passage, the death rate in currently developing nations has declined rapidly since World War II, while the birth rate has remained high with minor fluctuations. 4. It can be inferred from the passage that the first downward mortality trend (A) followed great fluctuations in fertility rates
44

http://tailieuso.com/

(B) followed great fluctuations in death rates (C) appeared after World War II (D) appeared before World War II (E) appeared around 1750 The best answer is E. According to the passage, no downward trend in mortality was noticeable before the middle of the eighteenth century.

Researchers criticize the transaction cost economics (TCE) paradigm for overgeneralizing the assumption of opportunism as part of human nature. However, some suggest that individualists have a higher opportunistic propensity in intragroup transactions, and collectivists in inter-group transactions. This cultural specification of opportunism helps TCE to accommodate more effectively some criticisms and more realistically deal with problems of economic organization in today's global economy. Transaction cost economics was first proposed by Coase and later popularized by Williamson. One of its key building blocks is the assumption of opportunism, because individuals, as Williamson claims, "will not reliably selfenforce promises but will defect from the letter and spirit of an agreement when it suits their purposes". Since, evidently, not all economic players are likely to be opportunistic, such a reliance on the assumption of opportunism has resulted in a torrent of criticisms, calling it dangerous, unhealthy, bad for practice, and an ethereal hand for organizational researchers. A primary reason TCE has provoked such a debate is precisely because it is centered on the assumption of opportunism, which touches on a fundamental question of human nature. To further develop this paradigm it is necessary to respond to the criticisms by clarifying and strengthening this important assumption. To be sure, TCE scholars never assumed that all (or most) individuals are likely to be opportunistic all (or most of) the time - a clearly indefensible position. Instead,
45

http://tailieuso.com/

most individuals are assumed to be "engaged in business-as-usual, with little or no thought to opportunism, most of the time". However, TCE suggests that it is the inability to differentiate opportunists, who may be a minority, from nonopportunists ex ante that necessitates the assumption of opportunism. TCEs critics, on the other hand, do not suggest that opportunism does not exist; rather, they caution against an over-reliance on the opportunism assumption because it may not be realistic to hold this assumption constant across individuals and organizations around the world. In order to make further theoretical progress, researchers must tackle the harder and more interesting issues of what kinds of individuals are likely to be opportunists, under what circumstances, and to what extent. Such an improved understanding of opportunism is important in today' s increasingly global economy, in which economic players from different backgrounds routinely interact with each other. 1. In the passage, the author is primarily concerned with doing which of the following? (A) Comparing two different approaches to a problem (B) Present a criticism of an assumption and suggesting how to clarify it (C) Describing a problem and proposing a solution (D) Presenting data and drawing conclusions from the data (E) Comparing two different analyses of a current situation The best answer is B. The passage presents criticism that TCEs assumption of opportunism has provoked and suggests what research should be carried out to resolve the issue. 2. It can be inferred from the passage that which of the following is true of TCE? (A) All economic players are likely to be opportunistic (B) The majority of economic players are opportunistic (C) The majority of human beings are opportunistic (D) Not all opportunists are opportunistic under the same circumstances (E) TCE has provoked such a debate because it is centered on the assumption of fundamentalism The best answer is D. According to the passage, TCE advocates claim that individualists have a higher opportunistic propensity in intra-group transactions, and collectivists in inter-group transactions. Thus, it can be inferred that not all opportunists act in an opportunistic fashion under the same circumstances 3. Which of the following would most logically be the topic of the paragraph immediately following the passage? (A) Specific ways to determine what kinds of individuals are likely to be opportunists, under what circumstances, and to what extent
46

http://tailieuso.com/

(B) The contributions of TCE to economic theory (C) Ways in which opportunists take advantage of unsuspecting business men (D) Nontraditional methods of testing individuals to find out if they are opportunists (E) The centrality of TSE to the position of opportunists in the business world The best answer is A. In the last paragraph of the passage it says that in order to make further theoretical progress researchers must tackle the issue of what kinds of individuals are likely to be opportunists, under what circumstances, and to what extent. It is logical that the following paragraph would do so. 4. The quotation in line ??? ["will not reliably self-enforce.] is most probably used to (A) counter a position that the author of the passage believes is correct (B) counter a position that the author of the passage believes is incorrect (C) elucidate a term (D) point out a paradox (E) present a historical maxim The best answer is C. The quotation is most likely used to explain what is meant by the term opportunism.

The mystery of cloud formation is as ancient as mankind. Throughout history, man prayed for rain and feared snow and lightning. Only in the last few decades, was man able to experiment and answer many questions as to how clouds are formed. Lately, it has seemed that no almost queries were left in that field. However, a recent look at old experimental data is threatening to overturn a longstanding theory about how water droplets freeze within clouds . Suspended water droplets can remain liquid even at temperatures far below the normal freezing point. Data collected in recent years show that clouds as cold as 37.5C can still contain many liquid droplets of water. Such droplets freeze solid almost instantly if they bump into each other or are otherwise disturbed . Most scientists have long assumed that a tiny globule of pure, liquid water, when disturbed, begins to freeze around an icy seed that suddenly forms inside
47

http://tailieuso.com/

it. According to this scenario, the time needed to freeze a given volume of water dispersed into a fine mist is independent of the size of the individual droplets because the formation of a seed particle is a chance event . However, the results of recent laboratory experiments, when combined with information gathered from tests conducted as many as 30 years ago, do not support this scenario. Together, the data indicate that the time needed to freeze a given volume of liquid water varies drastically according to droplet size. This extreme variation makes sense if freezing begins at the surface of the drops, not at the cores. Dividing a given volume of water into a large number of small droplets yields more total surface area than if the volume is split into a small number of large drops. The freezing rate would then depend on the surface area . The laws of thermodynamics also argue against ice nuclei forming inside liquid droplets. When water molecules begin to assemble into ice crystals, they release large amounts of latent heat. If that process had occurred in the center of a liquid droplet, the heat would have remained trapped within the globule, slowing the freezing process. But if crystallization begins at the droplet's surface, latent heat can more easily transfer to the surrounding air. In this case, the droplets are so cold that heat released internally as crystallization proceeds probably would not melt the developing ice . Familiar as clouds are, the behavior of their constituent droplets remains only partly understood. Lightning, rainfall, and other meteorological phenomena vary with the ratio of water droplets and ice particles in clouds. Linking freezing rates of clouds to those atmospheric and other climate processes is one of the most unreliable areas in current climate simulations. 1. According to the passage, if a given volume of water is divided into a large number of small droplets, as opposed to a smaller number or large droplets, it will freeze quicker. This indicates that (A) a given volume of water yields more droplets when below freezing (B) a given volume of water yields more droplets when above freezing (C) freezing begins simultaneously both at the cores and at the surface of the drops (D) freezing begins not at the surface of the drops, but at the cores (E) freezing begins not at the cores of the drops, but at the surface The best answer is E. If the relatively large amount of surface area provided by small droplets influences the time it takes to freeze, then freezing must begin on the surface of the droplets. 2. According to the passage, all of the following are characteristic of cloud droplets EXCEPT (A) they periodically bump into each other

48

http://tailieuso.com/

(B) the time needed to freeze a given volume of water dispersed into a fine mist is dependent on the size of the individual droplets (C) can remain liquid even at temperatures far below the normal freezing point (D) they are only partly understood by scientists (E) they appear in the same amount in clouds involved in various meteorological phenomena The best answer is E. According to the passage, the ratio of water droplets and ice particles in clouds varies in lightning, rainfall, and other meteorological phenomena 3. The passage suggests that many questions in the field of meteorology (A) have been answered (B) have no answers (C) have yet to be answered (D) have been answered incorrectly (E) are uninteresting to scientists today The best answer is A. The author states that it has lately seemed that no almost queries were left in meteorology. 4. The primary purpose of the passage is to (A) present several explanations for a well-known fact (B) argue in favor of a long-standing theory (C) discuss the implications of a new research finding (D) present new evidence that overturns a long-standing theory (E) question the methodology used in a study. The best answer is D. The passage presents new evidence on how water droplets freeze which overturns a long-standing theory that claims that droplets freeze from their center.

To unravel the extent to which Shakespeare is accorded cultural capital in India, two examples from popular culture will be examined - one from an Air India advertisement and another from a Hollywood film in the making that casts Bollywood star Hrithik Roshan as an Indian king in Rajasthan, looking to avenge the murder of his father.
49

http://tailieuso.com/

The Air India advertisement, found in a 1975 American magazine, features Shakespeare with a beautiful Indian woman clad in a sari. Focused from shoulders up, the black and white image reproduces a typical studio shot of a married couple. The top of the ad reads, "We go out of our way to please you. We also go to London every day." This is followed by the contact reference to Air India. Presented in the image of a couple, the ad naturalizes the union of the colonized Indian woman and Shakespeare (the most authoritative representative of the culture of the colonizer) and thrusts them into the global realm of tourism, art, and commerce to symbolize the union of East and West, with Shakespeare representing the means of transcending the distance between India and England. To make it more enticing, the slogan that follows the main caption of the ad is also somewhat sexualized: "We (Air India) work all day to make your night with us a dream." The advertisement creates a spectacle that reproduces Shakespeare in the image of the authoritative imperial traveler to the exotic and mysterious land, India, represented through the image of the exotic Indian woman. In so doing, the advertisement at once obscures the gendered, racial, and colonial dynamics that complicate such an image and legitimizes the discourse around Shakespeare as a harbinger of cultural authority . My second example is a Hollywood film in the making that plans to cast Indian Bollywood star Hrithik Roshan in the role of a Rajput prince who avenges his father's murder in a "chilling thriller." To generate publicity for the film, the filmmakers proposed to market Roshan as an "Indian Hamlet." While the producer believes that Hrithik is the ideal candidate for the role and places him "somewhere between Tom Cruise and Antonio Banderas," he recognizes that Hrithik is a new name in Hollywood. Therefore, he says, "we will have to sell him through the two known names--Hamlet and Shakespeare". In doing so, the Hollywood producer assumes the far-reaching influence of and appreciation for Shakespeare. At the same time, as a film that will be shot in India with some scenes in England, it conjures up the image of exoticism that continues to seize the West's imagination. Such images not only keep alive Shakespeares insidious influence in India, they further perpetuate the discourse about the bard's cultural superiority, which continues through government-sponsored agencies, the continuing presence of Shakespeare studies in education, and through theatre groups and touring companies performing locally and visiting from abroad. 1. According to the passage, Hrithik has to be marketed as Indias Hamlet because (A) his name is unknown to American audiences (B) American audiences confuse him with Tom Cruise and Antonio Banderas (C) the movies Hollywood producer assumes the far-reaching influence of and appreciation for Shakespeare
50

http://tailieuso.com/

(D) the film will be shot in India and England (E) Hrithik is the ideal candidate for the role The best answer is A. The author states that since Hrithik is a new name to American audiences he will have to be sold through two familiar names, Hamlet and Shakespeare. 2. The authors attitude toward the cultural capital accorded to Shakespeare in India is best described as one of (A) indifference (B) hesitance (C) disapproval (D) amusement (E) neutrality The best answer is C. In the last paragraph, the author refers to Shakespeares influence as insidious. 3. In can be inferred from the passage that more people in the U.S. are likely to pay to see the Hollywood movie mentioned in the passage if the actors are (A) Indian (B) British (C) Classically trained (D) familiar to the audience (E) excellent at portraying their roles The best answer is D. No information is given in the passage on how the factors mentioned in choices A, B, C, and E might influence a movie-goer. 4. In the last paragraph, the author is primarily concerned with (A) correcting an error occurring in one of the works under review (B) citing evidence to support a view of Shakespeare in India (C) drawing conclusions on the basis of evidence presented in the first three paragraphs (D) summarizing the arguments about Shakespeare presented in the first three paragraphs (E) refuting the view of Shakespeares influence in India presented in the previous paragraph The best answer is C. In the last paragraph, the author draws conclusions on the effects of the examples presented earlier in the passage.

51

http://tailieuso.com/

The relationship between science and religion, and even the one between skepticism and religion, is warming up. At least, that is the feeling one gets from a cursory look at recent happenings, including the proliferation of books and articles in popular magazines about science "finding" God. Thus, the time is ripe for a skeptical analysis of the subject, which seems muddled by two basic sources of confusion: the need to separate logical/philosophical arguments from those that are either pragmatic or concern freedom of speech; the need to acknowledge that there are many more possible positions on the science and religion question than are usually considered, and that a thorough understanding of the whole gamut is necessary to make any progress. The relationship between science and religion is a legitimate area of philosophical inquiry that must be informed by both theology and science . Discussions about science and religion, especially in the United States, carry practical consequences that do not affect both in an equal manner. Discussing science and religion has repercussions on the cherished value of freedom of speech for scientists, skeptics, and religionists . Attacks on religion are considered politically incorrect--the remarks by Minnesota Governor Jesse Ventura resulted in his popularity dropping 28 percent overnight. Scientists are especially aware of the fact that their research funding depends almost entirely on public financing through various federal agencies such as the National Science Foundation and the National Institutes of Health. Since federal funding is controlled by politicians, who in turn have a tendency to respond to every nuance of their constituency as gauged by the latest poll, it follows that no matter what your opinion as a scientist on matters of the spirit, it is wiser to stick to your job and avoid upsetting your benefactor . This is all the more so because of two other things we know about scientists: the overwhelming majority of them do not believe in a personal God (about 60% of general scientists and a staggering 93% of top scientists), and the reason they become scientists is to pursue questions for which science is a particularly good tool. Most of these questions are rather more mundane than the existence of God . The result of this odd mix is that while most prominent scientists do not believe in a personal God because of their understanding of science and of its implications, they must come out in public with conciliatory statements to the effect that there is no possible contradiction between the two. 1. The author mentions Governor Jesse Ventura (lines ???) most likely in order to
52

http://tailieuso.com/

(A) prove that the Governor does not believe in a personal God (B) show that the Governor believes in a personal God and therefore does not believe that various federal agencies such as the National Science Foundation and the National Institutes of Health should back scientific enquiry (C) show that the public is unlikely to finance a politician who denounces religion publicly (D) show that the public is unlikely to support a public figure who denounces religion publicly (E) show that the public is likely to back a public figure who denounces religion publicly The best answer is D. The author uses the governors case to show that if a public figure speaks out against religion that person would likely lose the support of the public 2. It can be inferred from the passage that the author believes that the questions asked by scientists are (A) more ordinary than questions about the existence of God (B) more extraordinary than questions about the existence of God (C) more popular than those asked by theologians (D) more popular than those asked by politicians (E) less extraordinary than those asked by politicians The best answer is A. The author states that science delves into questions that are more mundane than the existence of God. 3. The author implies that which of the following will occur if a scientist publicly declares that he does not believe in a personal God? (A) That scientist will likely lose his job. (B) That scientist will likely lose all support. (C) That scientist will likely lose federal support. (D) That scientist will likely lose all funding. (E) That scientist will likely lose federal funding. The best answer is E. According to the passage, a scientist that speaks out against a belief in God would likely lose federal funding because that funding is decided upon by politicians who do not want to lose the support of their voters. 4. The passage as a whole can be characterized as which of the following? (A) A description of an attitudinal change (B) A discussion of an analytical defect (C) A look at the interrelationship of two fields of inquiry (D) an argument in favor of revising a view (E) an evaluation of a scholarly study

53

http://tailieuso.com/

The best answer is C. The passage examines the relationship between science and religion.

The Technology-Related Assistance for Individuals with Disabilities Act of 1988 defines assistive technology devices as any item, piece of equipment, or product system, whether acquired commercially, modified, or customized, that is used to increase, maintain, or improve functional capabilities of individuals with disabilities. The promise offered by this law is that a student with a disability will be offered any assistive devices and services necessary to enhance his or her educational experience.

Although there is general agreement that technology is educationally beneficial, the research results have not strengthened the case for its impact on teaching children with disabilities. However, Edwards not only encourages an expanded use of technology in education, but also offer several reasons why its continued use is justified. For example, he believes that technologically based methods have promoted several motivational strategies such as gaining learner attention, getting learners to create their own technologically based products, and empowering learners to take control of their own learning. He also contends that technology can facilitate unique learning environments, make traditional learning environments more powerful and more effective by linking learners to information sources, help learners visualize problems and solutions, and connect learners to learning tools.

54

http://tailieuso.com/

While most agree that technology has obvious appeal for improving educational outcomes for students, teachers must ensure that it accommodates the specialized needs of students with disabilities. The diversified needs of students with various disabilities (e.g. behavior disorders, mental retardation, blind or vision impairment, deafness and so on) require different technological resources and applications. For example, assistive technology devices have been designed to bypass the need to type for learners with physical disabilities that effect their hands.

One should consider the following factors when selecting and using assistive devices. The user should ensure that the assistive device can be integrated into the student's instructional program, can endure rapid technological changes and, if warranted, can be upgraded, and can be easily maintained and that technical support, repair or maintenance is readily available. Additionally, school districts should ensure that teachers receive the necessary training and support for employing assistive devices and services so that teachers can provide students with the necessary motivation to use assistive devices.

55

http://tailieuso.com/

1. According to the passage, research on the use of technology in education (A) has not strengthened the contention that it should be used to educate children with disabilities (B) has not strengthened the contention that it should be used to educate children without disabilities (C) has not strengthened the contention that it should be used to educate deaf and hearing impaired children (D) has strengthened the contention that it should be used to educate deaf and hearing impaired children (E) has strengthened the contention that it should be used to educate physically handicapped children

The best answer is A. According to the author, although there is general agreement that technology is educationally beneficial, the research results have not strengthened the case for its impact on teaching children with disabilities.

2. According to Edwards which of the following is true about technologically bases methods (A) Technologically based methods make learning more enjoyable, making it possible to absorb more information. (B) Technologically based methods help disabled learners overcome their shyness by connecting them virtually with other non-disabled students (C) Technologically based methods shorten the learning curve for disabled students (D) Technologically based methods empower learners to take control of their own learning. (E) Technologically based methods circumvent to need for disabled student to get information for libraries that may be inaccessible.

56

http://tailieuso.com/

The best answer is D. In paragraph two, the author mentions students taking control of their own lives as one of the advantages of technologically based methods cited by Edwards.

3. According to the passage, all of the following can be considered assistive technology as defined by the Technology-Related Assistance for Individuals with Disabilities Act of 1988 EXCEPT (A) a product system used to improve functional capabilities of individuals with disabilities (B) a piece of equipment that has been modified to improve functional capabilities of individuals with disabilities (C) a piece of equipment that has been customized to replace the use of a limb in an individual with physical disabilities (D) a piece of equipment that has been modified to maintain functional capabilities of individuals with disabilities (E) a commercially acquired item used to increase functional capabilities of individuals with disabilities

The best answer is C. The passage mentions increasing, maintaining, improving, but not replacing functional capabilities of individuals with disabilities.

4. According to the passage, the value of a particular type of assistive technology depends on its ability to be which of the following? I Upgraded II Easily maintained III Link learners to information sources

(A) I only
57

http://tailieuso.com/

(B) II only (C) I and II only (D) I and III (E) I, II, and III

The best answer is C. In paragraph four, the author lists factors to consider before purchasing assistive software. I and II are included in the list. Economists have defined four stages societies pass through based on demographics, economics, agricultural productivity, and technological advancements. The first stage is a traditional society, characterized by low population density and low economic and population growth. In this society, which may be a country or a region, high birthrates match high death rates, while primitive technology contributes to low income and low living standards.

The second stage is a developing stage, occurring when a society's technological advancements result in sustainable agricultural production and plant and animal domestication. The result is a more plentiful food supply, which helps increase population growth mainly by slowing death rates. When population and food production growth are combined with industrialization and urbanization at this stage, the result is environmental exploitation and degradation present in many developing societies .

Increases in agricultural productivity and production bring economic surpluses that allow growth in capital and per capita income and the third stage, the developed society, is born. Birthrates fall faster than death rates as the roles of women change and developments in birth-control methods allow adults to choose the number of children they desire.

Finally, stage four, the mature society, sees notable technological change evolve beyond agriculture (particularly in medicine and public health), making
58

http://tailieuso.com/

death rates decline further. While many technological breakthroughs in developing societies come from innovative laypersons, breakthroughs in developed societies tend to require scarce, highly trained, experienced, and costly technicians and scientists. By the time the mature society develops, the most readily accessible raw materials have been exploited. Obsolescence of current technology requires investment in maintenance rather than in new technologies. Increase in productivity of service activities, which grow in importance, becomes more difficult than increases in agricultural and manufacturing productivity. Some developed regions choose to sacrifice some economic growth for equity. Thus, while productivity and income continue to rise, the rate of these increases slows. Rapid agricultural productivity gains continue in developed societies as investments in education and science made in the development stage produce long-term payoffs and as urbanization and industrialization lead to an exodus of agricultural labor. At the same time, slowing rates of income growth and population growth slow down the growth in demand for food. Food selfsufficiency increases in some countries after falling in the development stage. However, agricultural trade typically grows, as more affluent consumers demand a variety of foods from around the world .

Many developed countries have recently entered or will soon enter the fourth stage--the mature society--the future society of the world's inhabitants. It is a long-held view that global population growth will more or less stabilize; recent evidence, however, presents a strong case for negative global population growth as the seminal attribute of the mature society .

1. The primary purpose of the passage is to (A) discuss a plan for investigation of a process (B) summarize a theory and suggest a revision that overcomes a problem with the theory
59

http://tailieuso.com/

(C) describe a course of development (D) present a mechanism that causes change and development (E) challenge the validity of a theory by exposing inconsistencies

The best answer is C. The passage describes the development of societies from stage one to stage four.

2. It can be inferred from the passage that the birth rates in a third stage society is (A) the same as that of a fourth stage society (B) lower than that of a fourth stage society (C) the same as that of a second stage society (D) lower than that of a second stage society (E) faster than that of a second stage society The best answer is D. According to the passage, the birth rate in a third stage society falls faster than the death rate, which indicates that the rate is falling. Hence, the birth rate must be lower than that of a second stage society.

3. According to the passage, the categorization of societies into four stages outlined in the passage has been created by (A) industrialists (B) economists (C) politicians (D) the United Nations (E) sociologists

60

http://tailieuso.com/

The best answer is B. The first sentence of the passage starts: Economists have defined four stages .

4. According to recent evidence, negative global population growth is considered (A) a superfluous attribute of a mature society (B) a common aspiration of a mature society (C) the most important determining attribute of the mature society (D) the most important determining attribute of the developed society (E) a common myth of a developing society The best answer is C. The author states that negative global population is the seminal attribute of the mature society.

According to the United Nations, the total fertility rate - the number of children a woman may be expected to bear during her lifetime - has fallen in every region of the world since 1950. From an average of nearly six children per woman in the 1950s, total fertility rate fell to three children in Latin America, 3.4 in India, and 3.5 in other parts of Asia by the early to mid-1990s. The only major exception to this sustained downtrend is in North America, where the recent increase in total fertility rate appears to be a transitory phenomenon associated with immigration and a large number of baby-boom women deciding to have children relatively late in their lives.

As significant as declining total fertility rates worldwide is the fact that, from 1990 to 1995, the rates in Europe, China, and North America were below the
61

http://tailieuso.com/

2.1 average children per woman needed to sustain population worldwide over the long run. The United Nations' medium population projection of 2.1 children after 2040 is widely used as a demographic forecast, but it unrealistically assumes that this rate will be the same in both developed and developing countries. Many researchers, such as Wolfgang Lutz of the International Institute for Applied Systems Analysis, do not support the UN assertion that fertility would increase to replacement level in developed countries. Lutz and others cite evidence pointing toward low fertility, noting contraception, declining marriage rates, high divorce rates, increasing independence and career orientation of women, materialism, and consumerism. "These factors, together with increasing demands and personal expectations for attention, time, and also money to be given to children, are likely to result in fewer couples having more than one or two children and an increasing number of childless women," Lutz and his colleagues write in The Future Population of the World. The United Nations has a second scenario - the low/medium scenario - that presumes fertility averaging 1.9 children per woman for all regions by 2025. This scenario may be as unrealistic as the medium population scenario. The low/medium scenario may underestimate future total fertility rates in developing countries, just as the UN medium scenario may overestimate future total fertility rate in developed countries .

The low/medium scenario projects a peak world population of 7.9 billion people in 2050, declining to 6.4 billion by 2150. The medium scenario projects a peak world population of just less than 11 billion by approximately 2200. Most other projections, however, predict peak global population in less than a century, followed by negative population growth . These 1998 UN population figures were revised in 2000, and the new estimates, though tentative, indicate population trends even lower than the 1998 predictions. The low/medium scenario is comparable to what the United Nations now calls the low variant for worldwide populations. Given the low
62

http://tailieuso.com/

variant, the United Nations predicts that world population in 2050 will be 7.8 billion--slightly less than the 7.9 billion projected under the low/medium scenario. It is predicted that the United Nations will continue to revise population trends downward and that negative population growth will occur even sooner than 2150. However, because the 2000 population data remain tentative and do not extend to 2150, it is preferable to continue to rely on the 1998 data.

1. Which of the following statements can be inferred from the passage?

(A) immigrates to the United States give birth to fewer children than they would have, had they remained in their native countries (B) immigrates to the United States give birth to more children than they would have, had they remained in their native countries (C) more immigrates to the United States give birth to fewer children than they would have, had they remained in their native countries (D) less immigrates to the United States give birth to more children than they would have, had they remained in their native countries (E) immigrates to the United States give birth to fewer children, on average, than native born American woman

The best answer is E. Since elevated fertility rate can be partly accounted for by immigration, it can be inferred that the immigrants give birth to more children than do native Americans, on average.

2. It can be inferred from the passage that (A) the birth rate in South-East Asia has risen since the 1950s (B) the birth rate in Africa has declined since the 1950s (C) the birth rate in North America has declined since the 1950s
63

http://tailieuso.com/

(D) the birth rate in North America is higher than the death rate (E) the birth rate in North America is lower than the death rate

The best answer is B. According to the passage the fertility rate has declined in every region of the world except North America, therefore it can be inferred that the fertility rate dropped in Africa.

3. According to predictions made by Wolfgang Lutz, which of the following is likely to occur? (A) More women in future will remain childless than do so today. (B) More woman will put off having children until their thirties than do so today (C) More woman will give birth one or two babies than do today. (D) Less women will choose not to have children. (E) The number of children a couple has will be determined more and more often by the female partner. The best answer is A. According to the passage, Wolfgang Lutz predicts that various factors will lead to an increasing number of childless women.

4. According to the United Nations projected demographics, the low/medium forecast predicts (A) a peak in approximately 2050 while the medium forecast predicts a peak in approximately 2200 (B) a peak in approximately 2050 while the medium forecast predicts a peak in approximately 2150 (C) a peak in approximately 2150 while the medium forecast predicts a peak in approximately 2050 (D) a peak in approximately 2150 while the medium forecast predicts a peak in approximately 2200 (E) a peak in approximately 2200 while the medium forecast predicts a peak in approximately 2150
64

http://tailieuso.com/

The best answer is A. The low/medium forecast predicts that the population will reach its highest point 7.9 billion in approximately 2050. The medium forecast predicts that the population will reach its highest point 11 billion by approximately 2200. The racial identity of South Asians has long been a subject of some controversy in the United States. In the early years of the twentieth century, when whiteness, or African ancestry, was a prerequisite for naturalization, American courts vacillated on the question of whether Asian Indians were white or not. In contrast to Mexicans and Armenians, who were deemed white for the purposes of citizenship acquisition, and Japanese, Chinese, and Filipino applicants who were not, the verdict on the racial classification of Indians changed from case to case. American uncertainty over South Asian racial identity has also been mirrored in the Census Bureau's frequent changes in its classification of this group. Over the course of the last century, respondents of South Asian origin have been classified variously as `Hindu', `White', `Other', and `Asian'.

South Asian newcomers are not alone, however, in confronting an American racial landscape that at first seems to have no clear place for them. Not only does the diversity of the United States' contemporary immigrant pool ensure a steady influx of people who do not fit easily into the traditional black/white dichotomy, but in the past as well, immigrants tested, stretched and molded the nation's conceptions of racial categories. As Ignatiev has shown, Irish immigrants were not considered white until well after their arrival in the United States, and this was true of other European groups as well. Similarly, Chinese, Japanese, Korean, and Filipino Americans were not always considered to constitute a pan-ethnic Asian race.

But unlike the Irish who have already become white, or the Chinese and Japanese who are now Asian, the racial classification of South Asians in the United States is still in flux. Although they now seem firmly ensconced in the
65

http://tailieuso.com/

census `Asian' category, this is a recent development and one that came about only after considerable debate. Moreover, several writers have described an uneasy alliance between South Asians and East Asians under the pan-ethnic `Asian' rubric. Finally, other Americans seem unsure as to the racial status of these immigrants. F. James Davis finds evidence that some blacks consider Indians to be black as well, and Rosemary Marangoly George reports a widespread concern among Indian Americans in California over being taken for Mexican or black. More broadly, Nazli Kibria maintains that South Asians are seen as `ambiguous non-whites' in the United States .

Defined as `the socio-historical process by which racial categories are created, inhabited, transformed, and destroyed', racial formation is both a macro-level process and the culmination of myriad individual encounters. Given their inchoate racial status, South Asian Americans may offer unusual insight into this process of racial formation. 1. The main topic of the passage is (A) the socio-historical process by which racial categories are created (B) the racial status of ambiguous non-whites in the United States (C) census bureau classifications in the United States (D) the process of racial formation as exemplified by the case of the South Asians in the United States (E) the racial identity of South Asians in the United States

The best answer is E. Choices A and D are too general. The passage mentions ambiguous non-whites and census bureau classifications only as they relate to the identity of South Asians in the U.S.

2. The author mentions the Irish most likely in order to (A) prove how flexible racial categories can be over time
66

http://tailieuso.com/

(B) suggest that they may not really be white (C) counter the claim that all non-Africans are white (D) suggest that South Asians follow their example (E) differentiate between them and immigrants from China, Japan, Korea, and the Philippines

The best answer is A. Racial categories are considered by most to be fixed. The author gives the example of the Irish to prove that they can be flexible.

3. The information in the passage suggests that in the early twentieth century, it would have been difficult for a person from which of the following ethnic groups to obtain citizenship? (A) A Briton (B) A Filipino (C) A Japanese (D) An Armenian (E) An American of African descent The best answer is B. Britons and Armenians were considered white and so would be granted citizenship. Citizenship was also granted to people of African descent.

4. According to the author, which of the following is true of racial formation? (A) It is an irreversible process. (B) It is a social-historical process. (C) It offers unusual insight into South Asians. (D) It offers unusual insight into the human race. (E) It is a racist practice.

67

http://tailieuso.com/

The best answer is B. In paragraph four they author defines racial formation as the socio-historical process by which racial categories are created.

In parallel with advances in drug therapy, anesthesia, and surgery, there has been a shift in bioethics from a paternalistic ethic governed by doctors to one based on the patient's autonomy and integrity. The notion of consent to medical procedures and treatment is a reflection of this, and in many countries, this consent is now established in law. The terms "informed" and "patient's consent" were perhaps first coupled in the 1957 case of Salgo versus Leland Stanford Jr. University Board of Trustees. The plaintiff, paralyzed after myelography, had not been informed by his doctor that paralysis was a possible risk of this procedure. He won although the doctor had committed no mistake because the doctor failed in his duty of disclosure. The court found that if the patient had been properly informed he would have refused myelography.

From a legal perspective, any consent, if it is to be valid, has to meet three independent preconditions, all of which have to be taken into account simultaneously. Consent may be deemed invalid if it has been obtained by deception or coercion; if it does not comply with formal procedures; or if the person lacks the capacity to consent by virtue of mental illness. The weaker a patient's personal competence, the more stringent the procedural considerations must be. This is to avoid coercion or exploitation, and to ensure that the
68

http://tailieuso.com/

disclosure of information has taken into account the patient's capacity for understanding and evaluating the situation.

Equally, certain radical medical procedures call for stringent requirements to be met on personal and procedural competence. Such preconditions may be specifically formulated, as in Norway's abortion, sterilization, and transplantation legislation. These preconditions reflect a general principle of international health law, illustrated by the 1973 US case of Kaimowitz vs Michigan Department of Mental Health. Here the court found that even though a difficult psychiatric patient possessed the competence required to consent to ordinary surgical procedures, and even to "accepted neurosurgical procedures", his competence would not be sufficient for him to consent to experimental neurosurgical procedures characterized as "dangerous, intrusive, irreversible, and of uncertain benefit to the patient and the society".

The requirement for informed and voluntary consent is not always applicable to non-invasive medical procedures. In most cases, staff should be able to decide on the necessary procedures without having the patient's express consent, after having provided information to the patient. They must, however, respect a patient's rejection of any specific examinations or treatments. When the treatment offered is invasive, the doctor will have the responsibility for providing the patient with all necessary information--about the risks and the alternative treatments and their probable consequences.

1. According to the passage, when a treatment is invasive,

(A) difficult psychiatric patients cannot consent to ordinary surgical procedures, and even to "accepted neurosurgical procedures

69

http://tailieuso.com/

(B) difficult psychiatric patients must have a family member give written consent for any surgical procedures. (C) the patients express consent is necessary and it is the doctors responsibility to obtain it from the patient after he or she has received all the necessary information (D) the patients express consent is necessary and it is the staffs responsibility to obtain it from the patient after he or she has received all the necessary information (E) the patients express consent is if the patient has received all the necessary information about the risks and the alternative treatments and their probable consequences.

The best answer is C. Consent is necessary for invasive procedures, and it is the doctors responsibility to obtain it, not the staffs.

2. Which of the following, most accurately states the purpose of the passage? (A) To compare two different approaches to the question of consent. (B) To summarize two court cases regarding the question of informed consent. (C) To argue for a particular interpretation of the term informed consent. (D) To cite examples of how the notion of informed consent has been abused by medical staff working with psychiatric patients. (E) To discuss the notion of informed consent, its history and some variations on how the term is applied.

The best answer is E. The passage deals with various aspects of informed consent, including the history of the term and some special consideration to how it is applied.

70

http://tailieuso.com/

3. It can be inferred from the passage that prior to 1957 (A) doctors were allowed to continue performing the potentially dangerous myelography procedure (B) doctors were allowed to continue performing the potentially dangerous myelography procedure without the patients consent. (C) doctors were not allowed to neglect informing their staff of the dangers of a medical procedure and the alternatives that exist. (D) doctors were not required by law to inform their patients of the dangers of a medical procedure and the alternatives that exist. (E) doctors were required by law to inform their patients of the dangers of a medical procedure and the alternatives that exist.

The best answer is D. According to the passage, this case laid the precedent for informed consent, therefore, it can be inferred that consent was not required prior to the case.

4. Which of the following can be inferred from the passage about Norways abortion legislation?

(A) Abortion legislation in Norway requires that the patient sign a special consent form that is different from the general consent form required for other medical procedures. (B) Abortion legislation in Norway does not require that the patient sign a consent form because the patient is assumed to have personal and procedural competence. (C) Abortion legislation in Norway does require the patient to sign a consent form only when the attending physician believes the patient lacks personal and procedural competence. (D) Abortion legislation in Norway require the patient to sign a consent form only even when the attending physician believes the patient lacks personal and procedural competence.
71

http://tailieuso.com/

(E) Legislation in Norway requires the patient to sign a consent form only when a procedure is not intrusive so the law does not deal with abortion.

The best answer is A. According to the passage, the preconditions for abortion and other procedures are specially formulated.

Resuscitation after severe hemorrhage is a very delicate process that involves many biological procedures that need to be carried out within a very limited time window. To successfully revive a victim of severe exsanguination, one must work quickly within the "golden hour" time limit. In this time, there are various traumas that must be avoided; cardiac arrest, hypotension, hemorrhagic shock, and hypothermia to name a few.

72

http://tailieuso.com/

The resuscitation process involves the restoration of normotension (normative blood pressure) while avoiding cardiac arrest and hemorrhagic shock. Even when normotension has been restored, hemorrhagic shock, which can cause organ failure, can be a concern. There are three types of hemorrhagic shock: compensated hemorrhagic shock; uncompensated hemorrhagic shock, which is reversible; and irreversible hemorrhagic shock. The most common method of resuscitation makes use of lactated Ringer's solution to compensate for lost blood volume by causing the cells to swell, which in turn restores normotension. Though this works fairly well, it is not the optimal treatment.

Half of the deaths that occur annually are due to acute illness or injury, and are associated with circulatory failure or shock. Some of these deaths could be avoided by proper monitoring. Present technology involves monitoring early in the temporal course of an acute illness to observe the cardiac index, oxygen delivery and oxygen consumption. In future, a possibility for a very effective non-invasive monitoring device, would be one which could provide the following cardiac output readings: pulse oximetry for estimating arterial hemoglobin oxygen saturations, a reflection of pulmonary function; transcutaneous oxygen and CO2 tensions, reflections of tissue perfusion; and noninvasive blood pressure readings. Non-invasive systems are far more effective than invasive systems because they provide a constant display of the data and can be used at any location, whether in the hospital or in the field. In future, if systems like these are perfected, shock may be easily intercepted and avoided, thus resulting in a significant number of saved lives.

Future studies on resuscitation should be centered around shock, prevention of cardiac arrest as well as on increasing oxygen transport and increasing blood volume. The validity of blood pressure as a measure of organ viability and optimum possibility of resuscitation should be investigated as well. That said, resuscitation research should center mainly on developing procedures that can do

73

http://tailieuso.com/

all of these quickly and efficiently so that resuscitation will save lives everywhere from hospital to battlefield.

1. In this passage, the author is primarily interested in (A) describing the resuscitation process and suggesting strategies for improving it. (B) advocating particular strategies for future efforts to improve the resuscitation process (C) suggesting an alternative to an outdated research method (D) distinguishing between different resuscitation strategies (E) evaluating the effectiveness of resuscitation

The best answer is A. The passage both describes the resuscitation process and suggests how it may be improved in future.

2. The author mentions which of the following as a possible consequence of severe hemorrhage? (A) hypertension (B) normotension (C) increased blood volume (D) hemorrhagic shock (E) lactated Ringers solution

The best answer is D. Paragraph two of the passage discusses hemorrhagic shock, which, according to the passage, may occur after a severe hemorrhage even if normotension has been restored.

74

http://tailieuso.com/

3. According to the passage, which of the following statements is true about monitoring technology? (A) The present technology observes CO2 delivery and consumption, while future technology might provide readings on pulse oximetry and transcutaneous oxygen and CO2 tensions. (B) The present technology observes oxygen delivery and consumption, while future technology might provide readings on pulse oximetry and transcutaneous oxygen and CO2 tensions. (C) The present technology observes oxygen delivery and consumption, while future technology might provide readings on pulse oximetry and transcutaneous oxygen and carbon monoxide tensions. (D) The present technology observes oxygen delivery and consumption, while future technology might provide readings on pulse oximetry and transcutaneous oxygen and CO2 saturations. (E) The present technology regulates oxygen delivery and consumption, while future technology might evaluate pulse oximetry, transcutaneous oxygen and CO2 tensions. The best answer is B. The third paragraph outlines what the present technology now does, and what future technology may one day be able to do.

4. It can be inferred from the passage that after the golden hour has passed, (A) hypothermia will occur (B) there is no longer a chance of hypothermia (C) a patients chances of recovery are diminished (D) there is little or no chance that hemorrhagic shock will set in. (E) there is little or no chance of cardiac arrest.

The best answer is C. According to the passage, in order to successfully resuscitate a person suffering from severe hemorrhage, one must work within the golden hour time limit. In other words, after that time limit, the chances of recovery are lessened.

75

http://tailieuso.com/

Historians have traditionally focused on New England as the true birthplace of America, yet New England was fairly unrepresentative of the real America - a homogenous society dominated by English Puritans and their inflexible doctrines. The middle colonies, on the other hand, were made up of people of diverse origins, races, and creeds, so their interrelationships are unquestionably more symbolic of American culture. Like most people's idea of America, the middle colonies developed a commercial culture based on a balanced economy, and showed no real homogenous cultural traits that ran through the region. Indeed, most of the groups that coexisted in this region did not intermingle with each other, but kept their own distinctive cultural and social habits. Because of this, the argument can be made that the middle colonies were not the heterogeneous, "melting pot" culture that some historians claim existed. After all, heterogeneous seems to suggest a fusion of different types of people, when in fact these colonies offered more of a clannish type of policy when it came to dealing with their new neighbors. Jack Greene hypothesizes that the idea of mastery was a crucial factor in shaping America's colonial culture. Greene suggests that the objective of the English who migrated to the Americas was to achieve mastery over the rugged land of America as well as of other groups, a mastery that was unavailable to them in their homeland. The problem with this hypothesis is that it covers only the English migration to the New World, and only a relatively small portion of that group. In truth, many English chose to relocate to America for reasons having nothing to do with mastery over others, such as religious freedom and financial gain. Greene also advocates relating the culture of colonial America back to that of Great Britain, a technique that can be useful in delineating the differences between the two areas. However, taking this approach too far can be deleterious, as the English in America were quick to develop cultural traits that had little connection to their homeland. A comparison of some of these cultural aspects would only serve to confuse. Finally, the many people who came to America from countries besides England would not fit anywhere into Greene's paradigm.

1. The author mentions Jack Green's hypothesis most likely in order to


76

http://tailieuso.com/

(A) state a conclusion about facts presented in an earlier paragraph (B) qualify the extent to which a previously mentioned hypothesis is unsatisfactory (C) demonstrate that, of three explanatory factors offered, two may be incorrect (D) introduce a hypothesis that is considered unsatisfactory by the author (E) discredit a previously mentioned hypothesis. The best answer is D. In paragraph two of the passage, the author presents Greene's hypothesis along with reasons why it is not sufficient to understand the history of the America's colonial culture. 2. According to the author, the middle colonies were more representative of American colonial culture than New England because (A) the middle colonies were inhabited by Puritans while New England was inhabited by diverse creeds and cultures (B) the middle colonies had symbolic relationships while New England society was dominant (C) the middle colonies were peopled by a variety of people from different races and beliefs while New England was homogeneous (D) New England was peopled by a variety of people from different races and beliefs while the middle colonies were homogeneous (E) New England was inhabited by Puritans while the middle colonies were inhabited by diverse creeds and cultures

The best answer is C. According to the passage, New England was a homogenous society while the middle colonies had a heterogeneous mix of people. 3. Which of the following is NOT cited as an objective of some of the people that immigrated to America in colonial days? (A) (B) (C) (D) (E) Religious freedom Mastery over the land financial gain Mastery over other groups breaking out of a social class

The best answer is E. In paragraph two of the passage, the author cites A, B, C and D as possible reasons that individuals immigrated to America.

77

http://tailieuso.com/

4. The author's attitude to the claim that the middle colonies were a heterogeneous, "melting pot" culture is (A) limited approval (B) tentative acceptance (C) mild skepticism (D) studious criticism (E) wholehearted endorsement The best answer is D. The author rejects the claim that the middle colonies were a heterogeneous "melting pot" culture.

A computer virus is an illegal and potentially damaging computer program designed to infect other software by attaching itself to any software it contacts.
78

http://tailieuso.com/

In many cases, virus programs are designed to damage computer systems maliciously by destroying or corrupting data. If the infected software is transferred to or accessed by another computer system, the virus spreads to the other system. Viruses have become a serious problem in recent years, and currently, thousands of known virus programs exist. Viruses can be categorized as boot sector viruses, file viruses, and Trojan horse viruses. A boot sector virus infects the boot program used to start the system. When the infected boot program executes, the virus is loaded into the computer's memory. Once a virus is in the memory, it can spread to any floppy disk inserted into the computer. The second type of virus, a file virus, inserts viral code into program files. The virus then spreads to any program that accesses the infected file. A Trojan horse virus (named after a Greek myth) hides within or is designed to look like a legitimate program. Some viruses interrupt processing by freezing a computer system temporarily and then displaying sounds or messages. Other viruses contain time bombs or logic bombs. A time bomb is a program that performs an activity on a particular date. A logic bomb is a program that performs an activity when a certain action occurs, such as an employee being terminated. A worm, which is similar to a virus, copies itself repeatedly until no memory or disc space remains. To detect computer viruses, anti-virus programs have been developed. Besides detecting viruses, anti-virus programs also have utilities to remove or repair infected programs and files. Some damaged files cannot be repaired and must be replaced with uninfected backup files. Computers can best be protected by installing a virus protection software. In order to be effective, it is imperative to develop a regular plan for copying and storing important data and program files. Prior to running the virus protection software, floppy disks should be scanned using a virus scan. Since one of the most vulnerable points of entry for viruses is the internet, all programs and email messages downloaded from the Internet should be checked to make sure they are virus free.

1. The passage provides support for which of the following statements? A. Computer owners who use virus protection applications properly are more likely not to suffer damages from computer viruses. B. Computer owners who use virus protection applications properly will not suffer damages from computer viruses. C. Computer owners who use virus protection applications properly are more likely to suffer damages from computer viruses. D. Some damaged files can be repaired so must not be replaced with uninfected backup files.

79

http://tailieuso.com/

E. Some damaged files cannot be repaired and must be replaced with infected backup files. The best answer is A. The fourth paragraph states that computers can best be protected by installing virus protection software. 2. According to the passage, a worm is A. a virus that copies itself repeatedly until no memory or disc space remains. B. a virus that cannot be detected by an anti-virus program. C. something similar to a virus that works by copying itself repeatedly until no memory or disc space remains. D. something similar to a virus that uses a logic bomb to performs an activity when a certain action occurs. E. an anti-virus program that works by copying itself repeatedly until no memory or disc space remains. The best answer is C. Choices A and B are incorrect because a worm is not a virus. Choice D is incorrect because logic bomb are not used by worms. Choice E is incorrect because a worm is not an anti-virus program.

3. According to the passage, computer viruses can spread by which of the following means? I. Through boot programs II. Through floppy disks III. Through programs downloaded from the Internet A. I only B. II only C. I and II only D. II and III only E. I and II and III The best answer is D. Viruses can affect boot programs but they cannot spread through boot programs.

4. According to the passage, a virus that inserts viral code into program files is a A. Trojan horse virus B. time bomb C. worm D. file virus E. boot sector virus.
80

http://tailieuso.com/

The best answer is D. According to the second paragraph of the passage, a file virus inserts viral code into program files.

Until now, efforts at preventing the proliferation of bioweapons have relied on limiting access to the pathogens themselves. However, if terrorists can produce a bioweapon from genome information alone, the same logic suggests that access to genetic information should also be restricted. Scientists have begun to discuss the restriction of the export of critical pieces of DNA. This measure could be extended to the three big genome databases. Scientists have long been aware of the potential dangers. The creation of polio, for instance, was forecast months ago in an essay in Nature Immunology pointing out that it would be simple to build an artificial polio virus. Nevertheless, most scientists still oppose any attempts to restrict access to information. Earlier this year, the US Department of Defense dropped proposals for checking any research it funds for "sensitive" information before
81

http://tailieuso.com/

it is published, after scientists protested that this would impede research needed to defend against bioweapons. This was the case with anthrax. Unlike some other potential bioweapons, anthrax still exists in nature, infecting animals and sometimes humans, and samples are held in many labs worldwide. There is no reason for a terrorist to try to recreate it. The same is not true of Ebola, smallpox or the 1918 flu virus. Even so, when the American Society for Microbiology considered whether it should publish the smallpox genome, it reasoned that the benefits in terms of understanding the virus and designing drugs outweighed the risks. Not all scientists share such views. Raymond Zilinskas of the Monterey Institute of International Studies in California thinks some limits should be placed on the publication of information on organisms such as smallpox. Zilinskas contends that most scientists feel that basic research should not be restricted in this fashion. But where does one cross the line? He and others have proposed that professional societies and editorial boards at scientific journals should exert more control. Even if they do not, the decision could be taken out of their hands. In addition to international measures, individual governments are also cracking down. The USA Patriot Act passed this year allows the federal government to stop some foreign nationals working in the US from getting access to certain pathogens and toxins. The US could extend this to cover access to genetic sequences as well.

1. The passage is primarily concerned with (A) detailing the evidence that supports the claim that genetic information should be restricted (B) outlining the factors that have contributed to the restriction of genetic information (C) evaluating whether sensitive genetic information should be restricted in light of the danger of bioterrorism. (D) summarizing the differences between genetic research in free countries and such research in countries that promote terrorism. (E) formulating a hypothesis about the proliferation of bioweapons.

82

http://tailieuso.com/

The best answer is C. The passage does not reach any conclusions, it simply evaluates the question of limiting access to sensitive genetic information.

2. Terrorists would be unlikely to try to steal genetic information about anthrax because (A) it is difficult to control and could therefore be a threat to the terrorists themselves. (B) they do not have the necessary know-how to utilize genetic information to build anthrax powder (C) actual anthrax is not only available in nature, but also in laboratories. (D) the American Society for Microbiology has limited the dissemination of genetic information on anthrax. (E) the American Society for Microbiology has published genetic information on anthrax.

The best answer is C. Since anthrax itself is attainable, it is unlikely that bioterrorists would try to synthesize it using genetic information.

3. All of the following statements are supported by the passage EXCEPT: (A) the smallpox genome has been published by the American Society for Microbiology (B) federal government has stopped some foreign nationals from accessing to certain toxins (C) Raymond Zilinskas of the Monterey Institute believes the anthrax genome should not be published (D) Most scientists oppose any attempts to restrict access to information (E) it would not be difficult for a bioterrorist to build an artificial polio virus

83

http://tailieuso.com/

The best answer is C. According to the passage, Raymond Zilinskas believes some limits should be placed on the publication of information, but one cannot determine from the passage if that would include anthrax information.

4. It can be inferred from the passage that which of the following is true? (A) The USA Patriot Act will eventually stop all foreign workers from access to genomes. (B) It is not known whether or not bioterrorists have to skill to produce an actual bioweapon from genome information alone. (C) The Ebola virus affects animals. (D) According to Nature Immunology magazine, some terrorists have built an artificial polio virus (E) The three big genome databases have restricted of the export of critical pieces of DNA.

The best answer is B. In the first paragraph of the passage it is stated if terrorists can produce a bioweapon from genome information alone. One can infer from this that it is not known whether the terrorists can produce one or not.

84

Vous aimerez peut-être aussi